Programs & Examples On #Sigfpe

On POSIX-compliant platforms, SIGFPE is the signal sent to a process when it encounters an arithmetic error, such as division by zero.

A fatal error has been detected by the Java Runtime Environment: SIGSEGV, libjvm

Here is your relief for the problem :

I have a problem of running different versions of STS this morning, the application crash with the similar way as the question did.

Excerpt of my log file.

A fatal error has been detected by the Java Runtime Environment:
#a
#  SIGSEGV (0xb) at pc=0x00007f459db082a1, pid=4577, tid=139939015632640
#
# JRE version: 6.0_30-b12
# Java VM: Java HotSpot(TM) 64-Bit Server VM 
(20.5-b03 mixed mode linux-amd64 compressed oops)
# Problematic frame:
# C  [libsoup-2.4.so.1+0x6c2a1]  short+0x11

note that exception occured at # C [libsoup-2.4.so.1+0x6c2a1] short+0x11

Okay then little below the line:

R9 =0x00007f461829e550: <offset 0xa85550> in /usr/share/java/jdk1.6.0_30/jre/lib/amd64/server/libjvm.so at 0x00007f4617819000
R10=0x00007f461750f7c0 is pointing into the stack for thread: 0x00007f4610008000
R11=0x00007f459db08290: soup_session_feature_detach+0 in /usr/lib/x86_64-linux-gnu/libsoup-2.4.so.1 at 0x00007f459da9c000
R12=0x0000000000000000 is an unknown value
R13=0x000000074404c840 is an oop
{method} 

This line tells you where the actual bug or crash is to investigate more on this crash issue please use below links to see more, but let's continue the crash investigation and how I resolved it and the novelty of this bug :)

links are :

a fATAL ERROR JAVA THIS ONE IS GREAT LOTS OF USER!

a fATAL ERROR JAVA 2

Okay, after that here's what I found out to casue this case and why it happens as general advise.

  1. Most of the time, check that if you have installed, updated recently on Ubunu and Windows there are libraries like libsoup in linux which were the casuse of my crash.

  2. Check also for a new hardware problem and try to investigate the Logfile which STS or Java generated and also syslog in linux by

    tail - f /var/lib/messages or some other file
    

Then by carfully looking at those files the one you have the crash log for ... you can really solve the issue as follows:

sudo unlink /usr/lib/i386-linux-gnu/libsoup-2.4.so.1

or

sudo unlink /usr/lib/x86_64-linux-gnu/libsoup-2.4.so.1

Done !! Cheers!!

SQL Server - calculate elapsed time between two datetime stamps in HH:MM:SS format

DECLARE @EndTime AS DATETIME, @StartTime AS DATETIME

SELECT @StartTime = '2013-03-08 08:00:00', @EndTime = '2013-03-08 08:30:00'

SELECT CAST(@EndTime - @StartTime AS TIME)

Result: 00:30:00.0000000

Format result as you see fit.

Error: Cannot match any routes. URL Segment: - Angular 2

please modify your router.module.ts as:

const routes: Routes = [
{
    path: '',
    redirectTo: 'one',
    pathMatch: 'full'
},
{
    path: 'two',
    component: ClassTwo, children: [
        {
            path: 'three',
            component: ClassThree,
            outlet: 'nameThree',
        },
        {
            path: 'four',
            component: ClassFour,
            outlet: 'nameFour'
        },
        {
           path: '',
           redirectTo: 'two',
           pathMatch: 'full'
        }
    ]
},];

and in your component1.html

<h3>In One</h3>

<nav>
    <a routerLink="/two" class="dash-item">...Go to Two...</a>
    <a routerLink="/two/three" class="dash-item">... Go to THREE...</a>
    <a routerLink="/two/four" class="dash-item">...Go to FOUR...</a>
</nav>

<router-outlet></router-outlet>                   // Successfully loaded component2.html
<router-outlet name="nameThree" ></router-outlet> // Error: Cannot match any routes. URL Segment: 'three'
<router-outlet name="nameFour" ></router-outlet>  // Error: Cannot match any routes. URL Segment: 'three'

How to read request body in an asp.net core webapi controller?

The simplest possible way to do this is the following:

  1. In the Controller method you need to extract the body from, add this parameter: [FromBody] SomeClass value

  2. Declare the "SomeClass" as: class SomeClass { public string SomeParameter { get; set; } }

When the raw body is sent as json, .net core knows how to read it very easily.

php check if array contains all array values from another array

I think you're looking for the intersect function

array array_intersect ( array $array1 , array $array2 [, array $ ... ] )

array_intersect() returns an array containing all values of array1 that are present in all the arguments. Note that keys are preserved.

http://www.php.net/manual/en/function.array-intersect.php

How to make image hover in css?

Exact solution to your problem

You can change the image on hover by using content:url("YOUR-IMAGE-PATH");

For image hover use below line in your css:

img:hover

and to change the image on hover using the below config inside img:hover:

img:hover{
content:url("https://www.planwallpaper.com/static/images/9-credit-1.jpg");
}

Basic Python client socket example

Here is the simplest python socket example.

Server side:

import socket

serversocket = socket.socket(socket.AF_INET, socket.SOCK_STREAM)
serversocket.bind(('localhost', 8089))
serversocket.listen(5) # become a server socket, maximum 5 connections

while True:
    connection, address = serversocket.accept()
    buf = connection.recv(64)
    if len(buf) > 0:
        print buf
        break

Client Side:

import socket

clientsocket = socket.socket(socket.AF_INET, socket.SOCK_STREAM)
clientsocket.connect(('localhost', 8089))
clientsocket.send('hello')
  • First run the SocketServer.py, and make sure the server is ready to listen/receive sth
  • Then the client send info to the server;
  • After the server received sth, it terminates

Sass Nesting for :hover does not work

You can easily debug such things when you go through the generated CSS. In this case the pseudo-selector after conversion has to be attached to the class. Which is not the case. Use "&".

http://sass-lang.com/documentation/file.SASS_REFERENCE.html#parent-selector

.class {
    margin:20px;
    &:hover {
        color:yellow;
    }
}

How to add button tint programmatically

In addition to Shayne3000's answer you can also use a color resource (not only an int color). Kotlin version:

var indicatorViewDrawable = itemHolder.indicatorView.background
indicatorViewDrawable = DrawableCompat.wrap(indicatorViewDrawable)
val color = ResourcesCompat.getColor(context.resources, R.color.AppGreenColor, null) // get your color from resources
DrawableCompat.setTint(indicatorViewDrawable, color)
itemHolder.indicatorView.background = indicatorViewDrawable

cast a List to a Collection

Not knowing your code, it's a bit hard to answer your question, but based on all the info here, I believe the issue is you're trying to use Collections.sort passing in an object defined as Collection, and sort doesn't support that.

First question. Why is client defined so generically? Why isn't it a List, Map, Set or something a little more specific?

If client was defined as a List, Map or Set, you wouldn't have this issue, as then you'd be able to directly use Collections.sort(client).

HTH

BAT file to open CMD in current directory

As a more general solution you might want to check out the Microsoft Power Toy for XP that adds the "Open Command Window Here" option when you right-click: http://www.microsoft.com/windowsxp/downloads/powertoys/xppowertoys.mspx

In Vista and Windows 7, you'll get that option if you hold down shift and right-click (this is built in).

Emulate a 403 error page

To minimize the duty of the server make it simple:

.htaccess

ErrorDocument 403 "Forbidden"

PHP

header('HTTP/1.0 403 Forbidden');

die(); // or your message: die('Forbidden');

Compare two different files line by line in python

If order is preserved between files you might also prefer difflib. Although Rob?'s result is the bona-fide standard for intersections you might actually be looking for a rough diff-like:

from difflib import Differ

with open('cfg1.txt') as f1, open('cfg2.txt') as f2:
    differ = Differ()

    for line in differ.compare(f1.readlines(), f2.readlines()):
        if line.startswith(" "):
            print(line[2:], end="")

That said, this has a different behaviour to what you asked for (order is important) even though in this instance the same output is produced.

Tower of Hanoi: Recursive Algorithm

public static void hanoi(int number, String source, String aux, String dest)
{
    if (number == 1)
    {
        System.out.println(source + " - > "+dest);
    }
    else{
        hanoi(number -1, source, dest, aux);
        hanoi(1, source, aux, dest);
        hanoi(number -1, aux, source, dest);
    }
}

HorizontalScrollView within ScrollView Touch Handling

Thank you Joel for giving me a clue on how to resolve this problem.

I have simplified the code(without need for a GestureDetector) to achieve the same effect:

public class VerticalScrollView extends ScrollView {
    private float xDistance, yDistance, lastX, lastY;

    public VerticalScrollView(Context context, AttributeSet attrs) {
        super(context, attrs);
    }

    @Override
    public boolean onInterceptTouchEvent(MotionEvent ev) {
        switch (ev.getAction()) {
            case MotionEvent.ACTION_DOWN:
                xDistance = yDistance = 0f;
                lastX = ev.getX();
                lastY = ev.getY();
                break;
            case MotionEvent.ACTION_MOVE:
                final float curX = ev.getX();
                final float curY = ev.getY();
                xDistance += Math.abs(curX - lastX);
                yDistance += Math.abs(curY - lastY);
                lastX = curX;
                lastY = curY;
                if(xDistance > yDistance)
                    return false;
        }

        return super.onInterceptTouchEvent(ev);
    }
}

Where is Developer Command Prompt for VS2013?

For some reason, it doesn't properly add an icon when running Windows 8+. Here's how I solved it:

Using Windows Explorer, navigate to:

C:\ProgramData\Microsoft\Windows\Start Menu\Programs\Visual Studio 2013

In that folder, you'll see a shortcut named Visual Studio Tools that maps to (assuming default installation):

C:\Program Files (x86)\Microsoft Visual Studio 12.0\Common7\Tools\Shortcuts

Double-click the shortcut (or go to the folder above).

From that folder, copy the shortcut named Developer Command Prompt for VS2013 (and any others you find useful) to the first directory (for the Start Menu). You'll likely be prompted for administrative access (do so).

Once you've done that, you'll now have an icon available for the 2013 command prompt.

Using Image control in WPF to display System.Drawing.Bitmap

I wrote a program with wpf and used Database for showing images and this is my code:

SqlConnection con = new SqlConnection(@"Data Source=HITMAN-PC\MYSQL;
                                      Initial Catalog=Payam;
                                      Integrated Security=True");

SqlDataAdapter da = new SqlDataAdapter("select * from news", con);

DataTable dt = new DataTable();
da.Fill(dt);

string adress = dt.Rows[i]["ImgLink"].ToString();
ImageSource imgsr = new BitmapImage(new Uri(adress));
PnlImg.Source = imgsr;

What is mapDispatchToProps?

mapStateToProps receives the state and props and allows you to extract props from the state to pass to the component.

mapDispatchToProps receives dispatch and props and is meant for you to bind action creators to dispatch so when you execute the resulting function the action gets dispatched.

I find this only saves you from having to do dispatch(actionCreator()) within your component thus making it a bit easier to read.

https://github.com/reactjs/react-redux/blob/master/docs/api.md#arguments

How can I select an element by name with jQuery?

Frameworks usually use bracket names in forms, like:

<input name=user[first_name] />

They can be accessed by:

// in JS:
this.querySelectorAll('[name="user[first_name]"]')

// in jQuery:
$('[name="user[first_name]"]')

// or by mask with escaped quotes:
this.querySelectorAll("[name*=\"[first_name]\"]")

Translating touch events from Javascript to jQuery

$(window).on("touchstart", function(ev) {
    var e = ev.originalEvent;
    console.log(e.touches);
});

I know it been asked a long time ago, but I thought a concrete example might help.

R : how to simply repeat a command?

It's not clear whether you're asking this because you are new to programming, but if that's the case then you should probably read this article on loops and indeed read some basic materials on programming.

If you already know about control structures and you want the R-specific implementation details then there are dozens of tutorials around, such as this one. The other answer uses replicate and colMeans, which is idiomatic when writing in R and probably blazing fast as well, which is important if you want 10,000 iterations.

However, one more general and (for beginners) straightforward way to approach problems of this sort would be to use a for loop.

> for (ii in 1:5) { + print(ii) + } [1] 1 [1] 2 [1] 3 [1] 4 [1] 5 > 

So in your case, if you just wanted to print the mean of your Tandem object 5 times:

for (ii in 1:5) {     Tandem <- sample(OUT, size = 815, replace = TRUE, prob = NULL)     TandemMean <- mean(Tandem)     print(TandemMean) } 

As mentioned above, replicate is a more natural way to deal with this specific problem using R. Either way, if you want to store the results - which is surely the case - you'll need to start thinking about data structures like vectors and lists. Once you store something you'll need to be able to access it to use it in future, so a little knowledge is vital.

set.seed(1234) OUT <- runif(100000, 1, 2) tandem <- list() for (ii in 1:10000) {     tandem[[ii]] <- mean(sample(OUT, size = 815, replace = TRUE, prob = NULL)) }  tandem[1] tandem[100] tandem[20:25] 

...creates this output:

> set.seed(1234) > OUT <- runif(100000, 1, 2) > tandem <- list() > for (ii in 1:10000) { +     tandem[[ii]] <- mean(sample(OUT, size = 815, replace = TRUE, prob = NULL)) + } >  > tandem[1] [[1]] [1] 1.511923  > tandem[100] [[1]] [1] 1.496777  > tandem[20:25] [[1]] [1] 1.500669  [[2]] [1] 1.487552  [[3]] [1] 1.503409  [[4]] [1] 1.501362  [[5]] [1] 1.499728  [[6]] [1] 1.492798  >  

Mutex lock threads

A process consists of at least one thread (think of the main function). Multi threaded code will just spawn more threads. Mutexes are used to create locks around shared resources to avoid data corruption / unexpected / unwanted behaviour. Basically it provides for sequential execution in an asynchronous setup - the requirement for which stems from non-const non-atomic operations on shared data structures.

A vivid description of what mutexes would be the case of people (threads) queueing up to visit the restroom (shared resource). While one person (thread) is using the bathroom easing him/herself (non-const non-atomic operation), he/she should ensure the door is locked (mutex), otherwise it could lead to being caught in full monty (unwanted behaviour)

Android SDK location should not contain whitespace, as this cause problems with NDK tools

just change the path:

"c:\program files\android\sdk" to "c:\progra~1\android\sdk"
or
"c:\program files (x86)\android\sdk" to "c:\progra~2\android\sdk"

note that the paths should not contain spaces.

How to get root view controller?

Swift way to do it, you can call this from anywhere, it returns optional so watch out about that:

/// EZSwiftExtensions - Gives you the VC on top so you can easily push your popups
var topMostVC: UIViewController? {
    var presentedVC = UIApplication.sharedApplication().keyWindow?.rootViewController
    while let pVC = presentedVC?.presentedViewController {
        presentedVC = pVC
    }

    if presentedVC == nil {
        print("EZSwiftExtensions Error: You don't have any views set. You may be calling them in viewDidLoad. Try viewDidAppear instead.")
    }
    return presentedVC
}

Its included as a standard function in:

https://github.com/goktugyil/EZSwiftExtensions

How to extract the n-th elements from a list of tuples?

I know that it could be done with a FOR but I wanted to know if there's another way

There is another way. You can also do it with map and itemgetter:

>>> from operator import itemgetter
>>> map(itemgetter(1), elements)

This still performs a loop internally though and it is slightly slower than the list comprehension:

setup = 'elements = [(1,1,1) for _ in range(100000)];from operator import itemgetter'
method1 = '[x[1] for x in elements]'
method2 = 'map(itemgetter(1), elements)'

import timeit
t = timeit.Timer(method1, setup)
print('Method 1: ' + str(t.timeit(100)))
t = timeit.Timer(method2, setup)
print('Method 2: ' + str(t.timeit(100)))

Results:

Method 1: 1.25699996948
Method 2: 1.46600008011

If you need to iterate over a list then using a for is fine.

How to insert 1000 rows at a time

You can use the following CTE as well. You can just modify it as you find fit. But this will add the same values into the student CTE.

This will add 1000 records but you can change it to 10000 or to a maximum of 32767

;WITH thetable(rowid,sname,semail,spassword) AS
(
    SELECT 1  , 'name' , 'email' , 'password'
    UNION ALL
    SELECT rowid+1 ,'name' , 'email' , 'password' 
    FROM thetable WHERE rowid < 1000
)

SELECT rowid,sname,semail,spassword 
FROM thetable ORDER BY rowid
OPTION (MAXRECURSION 1000);

Laravel 5.2 - pluck() method returns array

The current alternative for pluck() is value().

Cannot get to $rootScope

I don't suggest you to use syntax like you did. AngularJs lets you to have different functionalities as you want (run, config, service, factory, etc..), which are more professional.In this function you don't even have to inject that by yourself like

MainCtrl.$inject = ['$scope', '$rootScope', '$location', 'socket', ...];

you can use it, as you know.

android set button background programmatically

Using setBackgroundColor() affects the style. So, declare a new style of the same properties with respect to the previous button, with a a different color.

<?xml version="1.0" encoding="utf-8"?>
<shape xmlns:android="http://schemas.android.com/apk/res/android"
android:shape="rectangle">
<solid android:color="@color/green"/>
<corners android:radius="10dp"/>
</shape>

Now, use OnClick method.

location.setOnClickListener(new View.OnClickListener() {
        @Override
        public void onClick(View v) {

            location.setBackgroundResource(R.drawable.green);

        }
    });

this changes the button but looks similar to changing the background.

How to run a .awk file?

If you put #!/bin/awk -f on the first line of your AWK script it is easier. Plus editors like Vim and ... will recognize the file as an AWK script and you can colorize. :)

#!/bin/awk -f
BEGIN {}  # Begin section
{}        # Loop section
END{}     # End section

Change the file to be executable by running:

chmod ugo+x ./awk-script

and you can then call your AWK script like this:

`$ echo "something" | ./awk-script`

iTerm2 keyboard shortcut - split pane navigation

I was using Terminator before, so I found it convenient to re-map Alt + arrow-key to switch between the panes. This can be done in Preferences -> Keys -> Key Mappings - press the '+' button to add a mapping. Also, in my case such a mapping was already defined in Profiles, I simply removed it.

Sort dataGridView columns in C# ? (Windows Form)

You can control the data returned from SQL database by ordering the data returned:

orderby [Name]

If you execute the SQL query from your application, order the data returned. For example, make a function that calls the procedure or executes the SQL and give it a parameter that gets the orderby criteria. Because if you ordered the data returned from database it will consume time but order it since it's executed as you say that you want it to be ordered not from the UI you want it to be ordered in the run time so order it when executing the SQL query.

How to edit an Android app?

Generally speaking, a software product isn't your "property already", as you said in the comment. Most of the times (I won't be irresponsible to say anything in open), it's licensed to you. A license to use some thing is not the same thing as owning (property rights) that very same thing.

That's because there are authorship, copyright, intellectual property rights applicable to it. I don't know how things work in United States (or in your country), but it's generally accepted that the work of a mind, a creative work, must not be changed in its nature as such to make the expression of art to be different than that expression that the author intended. That applies for example, in some cases, to architectural work (in most countries, you can't change the appearance of a building to "desfigure" the work of art of the architect, without his prior consent). Exceptions are made, obviously, when the author expressly authorizes such changes (e.g., Creative Commons licenses, open source licenses etc.).

Anyway, that's why you see in most EULAs the typical sentence: "this software is licensed, not sold". That's the purpose and reason why.

Now that you understand the reasons why you can't wander around changing other people's art, let me be technical.

There are possible ways to decompile Java programs. You can use dex2jar, it provides a somewhat good start for you to start looking for things and changes. And perhaps rebuild the code by mounting back the pieces together. Good luck, as most people obfuscate their codes to make that harder.

However, let me say that it's still forbidden to change programs, as I said above. And it's extremely unethical. It makes me sad that people do that with no scruples (not saying it's your case, just warning you). It shouldn't need people to be at the other side to understand that. Or maybe that's just me, who lives in a country where piracy is rampant.

The tools are always out there. But the conscience, unfortunately, not always.

edit: in case it isn't clear enough already, I do NOT approve the use of these programs. I use them myself to check how hard my own applications are to be reverse engineered. But I also think that explaning is always better than denial (better be here).

Unit testing private methods in C#

In VS 2005/2008 you can use private accessor to test private member,but this way was disappear in later version of VS

eloquent laravel: How to get a row count from a ->get()

Its better to access the count with the laravels count method

$count = Model::where('status','=','1')->count();

or

$count = Model::count();

Is there an equivalent of lsusb for OS X

On Mac OS X, the Xcode developer suite includes the USB Proper.app application. This is found in /Developer/Applications/Utilities/. USB Prober will allow you to examine the device and interface descriptors.

javascript remove "disabled" attribute from html input

Why not just remove that attribute?

  1. vanilla JS: elem.removeAttribute('disabled')
  2. jQuery: elem.removeAttr('disabled')

Using the star sign in grep

'*' works as a modifier for the previous item. So 'abc*def' searches for 'ab' followed by 0 or more 'c's follwed by 'def'.

What you probably want is 'abc.*def' which searches for 'abc' followed by any number of characters, follwed by 'def'.

angular 4: *ngIf with multiple conditions

<div *ngIf="currentStatus !== ('status1' || 'status2' || 'status3' || 'status4')">

Git push error pre-receive hook declined

ihave followed the instruction of heroku logs img https://devcenter.heroku.com/articles/buildpacks#detection-failure ( use cmd: heroku logs -> show your error) then do the cmd: "heroku buildpacks:clear". finally, it worked for me!

Visual Studio Code cannot detect installed git

In Visual Studio Code open 'user settings': ctrl + p and type >sett press enter

This will open default settings on left side and User settings on right side.

Just add path to git.exe in user settings

"git.path": "C:\\Users\\[WINDOWS_USER]\\AppData\\Local\\Programs\\Git\\bin\\git.exe"

Replace [WINDOWS_USER] with your user name.

Restart Visual Studio Code

SyntaxError of Non-ASCII character

You should define source code encoding, add this to the top of your script:

# -*- coding: utf-8 -*-

The reason why it works differently in console and in the IDE is, likely, because of different default encodings set. You can check it by running:

import sys
print sys.getdefaultencoding()

Also see:

Getting Hour and Minute in PHP

In addressing your comment that you need your current time, and not the system time, you will have to make an adjustment yourself, there are 3600 seconds in an hour (the unit timestamps use), so use that. for example, if your system time was one hour behind:

$time = date('H:i',time() + 3600);

How to record phone calls in android?

Ok, for this first of all you need to use Device Policy Manager, and need to make your device Admin device. After that you have to create one BroadCast receiver and one service. I am posting code here and its working fine.

MainActivity:

public class MainActivity extends Activity {
    private static final int REQUEST_CODE = 0;
    private DevicePolicyManager mDPM;
    private ComponentName mAdminName;

    @Override
    protected void onCreate(Bundle savedInstanceState) {
        super.onCreate(savedInstanceState);
        setContentView(R.layout.activity_main);

        try {
            // Initiate DevicePolicyManager.
            mDPM = (DevicePolicyManager) getSystemService(Context.DEVICE_POLICY_SERVICE);
            mAdminName = new ComponentName(this, DeviceAdminDemo.class);

            if (!mDPM.isAdminActive(mAdminName)) {
                Intent intent = new Intent(DevicePolicyManager.ACTION_ADD_DEVICE_ADMIN);
                intent.putExtra(DevicePolicyManager.EXTRA_DEVICE_ADMIN, mAdminName);
                intent.putExtra(DevicePolicyManager.EXTRA_ADD_EXPLANATION, "Click on Activate button to secure your application.");
                startActivityForResult(intent, REQUEST_CODE);
            } else {
                // mDPM.lockNow();
                // Intent intent = new Intent(MainActivity.this,
                // TrackDeviceService.class);
                // startService(intent);
            }
        } catch (Exception e) {
            e.printStackTrace();
        } 
    }

    @Override
    protected void onActivityResult(int requestCode, int resultCode, Intent data) {
        super.onActivityResult(requestCode, resultCode, data);

        if (REQUEST_CODE == requestCode) {
                Intent intent = new Intent(MainActivity.this, TService.class);
                startService(intent);
        }
    }

}

//DeviceAdminDemo class

public class DeviceAdminDemo extends DeviceAdminReceiver {
    @Override
    public void onReceive(Context context, Intent intent) {
        super.onReceive(context, intent);
    }

    public void onEnabled(Context context, Intent intent) {
    };

    public void onDisabled(Context context, Intent intent) {
    };
}

//TService Class

public class TService extends Service {
    MediaRecorder recorder;
    File audiofile;
    String name, phonenumber;
    String audio_format;
    public String Audio_Type;
    int audioSource;
    Context context;
    private Handler handler;
    Timer timer;
    Boolean offHook = false, ringing = false;
    Toast toast;
    Boolean isOffHook = false;
    private boolean recordstarted = false;

    private static final String ACTION_IN = "android.intent.action.PHONE_STATE";
    private static final String ACTION_OUT = "android.intent.action.NEW_OUTGOING_CALL";
    private CallBr br_call;




    @Override
    public IBinder onBind(Intent arg0) {
        // TODO Auto-generated method stub
        return null;
    }

    @Override
    public void onDestroy() {
        Log.d("service", "destroy");

        super.onDestroy();
    }

    @Override
    public int onStartCommand(Intent intent, int flags, int startId) {
        // final String terminate =(String)
        // intent.getExtras().get("terminate");//
        // intent.getStringExtra("terminate");
        // Log.d("TAG", "service started");
        //
        // TelephonyManager telephony = (TelephonyManager)
        // getSystemService(Context.TELEPHONY_SERVICE); // TelephonyManager
        // // object
        // CustomPhoneStateListener customPhoneListener = new
        // CustomPhoneStateListener();
        // telephony.listen(customPhoneListener,
        // PhoneStateListener.LISTEN_CALL_STATE);
        // context = getApplicationContext();

        final IntentFilter filter = new IntentFilter();
        filter.addAction(ACTION_OUT);
        filter.addAction(ACTION_IN);
        this.br_call = new CallBr();
        this.registerReceiver(this.br_call, filter);

        // if(terminate != null) {
        // stopSelf();
        // }
        return START_NOT_STICKY;
    }

    public class CallBr extends BroadcastReceiver {
        Bundle bundle;
        String state;
        String inCall, outCall;
        public boolean wasRinging = false;

        @Override
        public void onReceive(Context context, Intent intent) {
            if (intent.getAction().equals(ACTION_IN)) {
                if ((bundle = intent.getExtras()) != null) {
                    state = bundle.getString(TelephonyManager.EXTRA_STATE);
                    if (state.equals(TelephonyManager.EXTRA_STATE_RINGING)) {
                        inCall = bundle.getString(TelephonyManager.EXTRA_INCOMING_NUMBER);
                        wasRinging = true;
                        Toast.makeText(context, "IN : " + inCall, Toast.LENGTH_LONG).show();
                    } else if (state.equals(TelephonyManager.EXTRA_STATE_OFFHOOK)) {
                        if (wasRinging == true) {

                            Toast.makeText(context, "ANSWERED", Toast.LENGTH_LONG).show();

                            String out = new SimpleDateFormat("dd-MM-yyyy hh-mm-ss").format(new Date());
                            File sampleDir = new File(Environment.getExternalStorageDirectory(), "/TestRecordingDasa1");
                            if (!sampleDir.exists()) {
                                sampleDir.mkdirs();
                            }
                            String file_name = "Record";
                            try {
                                audiofile = File.createTempFile(file_name, ".amr", sampleDir);
                            } catch (IOException e) {
                                e.printStackTrace();
                            }
                            String path = Environment.getExternalStorageDirectory().getAbsolutePath();

                            recorder = new MediaRecorder();
//                          recorder.setAudioSource(MediaRecorder.AudioSource.VOICE_CALL);

                            recorder.setAudioSource(MediaRecorder.AudioSource.VOICE_COMMUNICATION);
                            recorder.setOutputFormat(MediaRecorder.OutputFormat.AMR_NB);
                            recorder.setAudioEncoder(MediaRecorder.AudioEncoder.AMR_NB);
                            recorder.setOutputFile(audiofile.getAbsolutePath());
                            try {
                                recorder.prepare();
                            } catch (IllegalStateException e) {
                                e.printStackTrace();
                            } catch (IOException e) { 
                                e.printStackTrace();
                            }
                            recorder.start();
                            recordstarted = true;
                        }
                    } else if (state.equals(TelephonyManager.EXTRA_STATE_IDLE)) {
                        wasRinging = false;
                        Toast.makeText(context, "REJECT || DISCO", Toast.LENGTH_LONG).show();
                        if (recordstarted) {
                            recorder.stop();
                            recordstarted = false;
                        }
                    }
                }
            } else if (intent.getAction().equals(ACTION_OUT)) {
                if ((bundle = intent.getExtras()) != null) {
                    outCall = intent.getStringExtra(Intent.EXTRA_PHONE_NUMBER);
                    Toast.makeText(context, "OUT : " + outCall, Toast.LENGTH_LONG).show();
                }
            }
        }
    }

}

//Permission in manifest file

<uses-permission android:name="android.permission.WRITE_EXTERNAL_STORAGE" />
    <uses-permission android:name="android.permission.RECORD_AUDIO" />
    <uses-permission android:name="android.permission.PROCESS_OUTGOING_CALLS" />
    <uses-permission android:name="android.permission.READ_PHONE_STATE" />
    <uses-permission android:name="android.permission.STORAGE" />

//my_admin.xml

<device-admin xmlns:android="http://schemas.android.com/apk/res/android" >
    <uses-policies>
        <force-lock />
    </uses-policies>
</device-admin>

//Declare following thing in manifest:

Declare DeviceAdminDemo class to manifest:

 <receiver
            android:name="com.example.voicerecorder1.DeviceAdminDemo"
            android:description="@string/device_description"
            android:label="@string/device_admin_label"
            android:permission="android.permission.BIND_DEVICE_ADMIN" >
            <meta-data
                android:name="android.app.device_admin"
                android:resource="@xml/my_admin" />


            <intent-filter>
                <action android:name="android.app.action.DEVICE_ADMIN_ENABLED" />
                <action android:name="android.app.action.DEVICE_ADMIN_DISABLED" />
                <action android:name="android.app.action.DEVICE_ADMIN_DISABLE_REQUESTED" />
            </intent-filter>
        </receiver>

<service android:name=".TService" >
        </service>

Best way to center a <div> on a page vertically and horizontally?

_x000D_
_x000D_
div {_x000D_
    position: absolute;_x000D_
    top: 50%;_x000D_
    left: 50%;_x000D_
    transform: translate(-50%, -50%);_x000D_
    -ms-transform: translate(-50%, -50%); /* IE 9 */_x000D_
    -webkit-transform: translate(-50%, -50%); /* Chrome, Safari, Opera */     _x000D_
}
_x000D_
<body>_x000D_
    <div>Div to be aligned vertically</div>_x000D_
</body>
_x000D_
_x000D_
_x000D_

position: absolute div in body document

An element with position: absolute; is positioned relative to the nearest positioned ancestor (instead of positioned relative to the viewport (body tag), like fixed).

However; if an absolute positioned element has no positioned ancestors, it uses the document body, and moves along with page scrolling.

source: CSS position

HTML Table cell background image alignment

use like this your inline css

<td width="178" rowspan="3" valign="top" 
align="right" background="images/left.jpg" 
style="background-repeat:background-position: right top;">
</td>

How to get current date & time in MySQL?

You can use not only now(), also current_timestamp() and localtimestamp(). The main reason of incorrect display timestamp is inserting NOW() with single quotes! It didn't work for me in MySQL Workbench because of this IDE add single quotes for mysql functions and i didn't recognize it at once )

Don't use functions with single quotes like in MySQL Workbench. It doesn't work.

How to update array value javascript?

Why not use an object1?

var dict = { "a": 1, "b": 2, "c": 3 };

Then you can update it like so

dict.a = 23;

or

dict["a"] = 23;

If you wan't to delete2 a particular key, it's as simple as:

delete dict.a;

1 See Objects vs arrays in Javascript for key/value pairs.
2 See the delete operator.

What is the equivalent of bigint in C#?

if you are using bigint in your database table, you can use Long in C#

Adding null values to arraylist

You could create Util class:

public final class CollectionHelpers {
    public static <T> boolean addNullSafe(List<T> list, T element) {
        if (list == null || element == null) {
            return false;
        }

        return list.add(element);
    }
}

And then use it:

Element element = getElementFromSomeWhere(someParameter);
List<Element> arrayList = new ArrayList<>();
CollectionHelpers.addNullSafe(list, element);

How to tell bash that the line continues on the next line

\ does the job. @Guillaume's answer and @George's comment clearly answer this question. Here I explains why The backslash has to be the very last character before the end of line character. Consider this command:

   mysql -uroot \
   -hlocalhost      

If there is a space after \, the line continuation will not work. The reason is that \ removes the special meaning for the next character which is a space not the invisible line feed character. The line feed character is after the space not \ in this example.

Regular Expression to match only alphabetic characters

This will match one or more alphabetical characters:

/^[a-z]+$/

You can make it case insensitive using:

/^[a-z]+$/i

or:

/^[a-zA-Z]+$/

how to create insert new nodes in JsonNode?

I've recently found even more interesting way to create any ValueNode or ContainerNode (Jackson v2.3).

ObjectNode node = JsonNodeFactory.instance.objectNode();

Create a CSV File for a user in PHP

To have it send it as a CSV and have it give the file name, use header():

http://us2.php.net/header

header('Content-type: text/csv');
header('Content-disposition: attachment; filename="myfile.csv"');

As far as making the CSV itself, you would just loop through the result set, formatting the output and sending it, just like you would any other content.

Plot multiple columns on the same graph in R

To select columns to plot, I added 2 lines to Vincent Zoonekynd's answer:

#convert to tall/long format(from wide format)
col_plot = c("A","B")
dlong <- melt(d[,c("Xax", col_plot)], id.vars="Xax")  

#"value" and "variable" are default output column names of melt()
ggplot(dlong, aes(Xax,value, col=variable)) +
  geom_point() + 
  geom_smooth()

Google "tidy data" to know more about tall(or long)/wide format.

How can I convert a long to int in Java?

If direct casting shows error you can do it like this:

Long id = 100;
int int_id = (int) (id % 100000);

Class vs. static method in JavaScript

I use namespaces:

var Foo = {
     element: document.getElementById("id-here"),

     Talk: function(message) {
            alert("talking..." + message);
     },

     ChangeElement: function() {
            this.element.style.color = "red";
     }
};

And to use it:

Foo.Talk("Testing");

Or

Foo.ChangeElement();

How to randomize Excel rows

Here's a macro that allows you to shuffle selected cells in a column:

Option Explicit

Sub ShuffleSelectedCells()
  'Do nothing if selecting only one cell
  If Selection.Cells.Count = 1 Then Exit Sub
  'Save selected cells to array
  Dim CellData() As Variant
  CellData = Selection.Value
  'Shuffle the array
  ShuffleArrayInPlace CellData
  'Output array to spreadsheet
  Selection.Value = CellData
End Sub

Sub ShuffleArrayInPlace(InArray() As Variant)
''''''''''''''''''''''''''''''''''''''''''''''''''''''''''''''''''''''''''''''''''''
' ShuffleArrayInPlace
' This shuffles InArray to random order, randomized in place.
' Source: http://www.cpearson.com/excel/ShuffleArray.aspx
' Modified by Tom Doan to work with Selection.Value two-dimensional arrays.
''''''''''''''''''''''''''''''''''''''''''''''''''''''''''''''''''''''''''''''''''''
  Dim J As Long, _
    N As Long, _
    Temp As Variant
  'Randomize
  For N = LBound(InArray) To UBound(InArray)
    J = CLng(((UBound(InArray) - N) * Rnd) + N)
    If J <> N Then
      Temp = InArray(N, 1)
      InArray(N, 1) = InArray(J, 1)
      InArray(J, 1) = Temp
    End If
  Next N
End Sub

You can read the comments to see what the macro is doing. Here's how to install the macro:

  1. Open the VBA editor (Alt + F11).
  2. Right-click on "ThisWorkbook" under your currently open spreadsheet (listed in parentheses after "VBAProject") and select Insert / Module.
  3. Paste the code above and save the spreadsheet.

Now you can assign the "ShuffleSelectedCells" macro to an icon or hotkey to quickly randomize your selected rows (keep in mind that you can only select one column of rows).

How to convert a Collection to List?

Here is a sub-optimal solution as a one-liner:

Collections.list(Collections.enumeration(coll));

Get the client IP address using PHP

In PHP 5.3 or greater, you can get it like this:

$ip = getenv('HTTP_CLIENT_IP')?:
getenv('HTTP_X_FORWARDED_FOR')?:
getenv('HTTP_X_FORWARDED')?:
getenv('HTTP_FORWARDED_FOR')?:
getenv('HTTP_FORWARDED')?:
getenv('REMOTE_ADDR');

Binding multiple events to a listener (without JQuery)?

I have a simpler solution for you:

window.onload = window.onresize = (event) => {
    //Your Code Here
}

I've tested this an it works great, on the plus side it's compact and uncomplicated like the other examples here.

Selecting multiple items in ListView

and to get it :

public void onItemClick(AdapterView<?> parent, View view,
                int position, long id) {

                Log.d(getLocalClassName(), "onItemClick(" + view + ","
                        + position + "," + id + ")");
        }
    });

How do you enable auto-complete functionality in Visual Studio C++ express edition?

Have you tried Visual Assist X ? Sort of lights up the VS editor.

event.preventDefault() vs. return false

The main difference between return false and event.preventDefault() is that your code below return false will not be executed and in event.preventDefault() case your code will execute after this statement.

When you write return false it do the following things for you behind the scenes.

* Stops callback execution and returns immediately when called.
* event.stopPropagation();
* event.preventDefault();

LaTeX package for syntax highlighting of code in various languages

You can use the listings package. It supports many different languages and there are lots of options for customising the output.

\documentclass{article}
\usepackage{listings}

\begin{document}
\begin{lstlisting}[language=html]
<html>
    <head>
        <title>Hello</title>
    </head>
    <body>Hello</body>
</html>
\end{lstlisting}
\end{document}

Iframe positioning

It's because you're missing position:relative; on #contentframe

<div id="contentframe" style="position:relative; top: 160px; left: 0px;">

position:absolute; positions itself against the closest ancestor that has a position that is not static. Since the default is static that is what was causing your issue.

C compiling - "undefined reference to"?

As stated by a few others, this is a linking error. The section of code where this function is being called doesn't know what this function is. It either needs to be declared in a header file an defined in its own source file, or defined or declared in the same source file, above where it's being called.

Edit: In older versions of C, C89/C90, function declarations weren't actually required. So, you could just add the definition anywhere in the file in which you're using the function, even after the call and the compiler would infer the declaration. For example,

int main()
{
  int a = func();
}

int func()
{
   return 1;
}

However, this isn't good practice today and most languages, C++ for example, won't allow it. One way to get away with defining the function in the same source file in which you're using it, is to declare it at the beginning of the file. So, the previous example would look like this instead.

int func();

int main()
{
   int a = func();
}

int func()
{
  return 1;
}

How to safely upgrade an Amazon EC2 instance from t1.micro to large?

Using the AWS Management Console

  • Go to "Volumes" and create a Snapshot of your instance's volume.
  • Go to "Snapshots" and select "Create Image from Snapshot".
  • Go to "AMIs" and select "Launch Instance" and choose your "Instance Type" etc.

Why doesn't Java offer operator overloading?

Assuming you wanted to overwrite the previous value of the object referred to by a, then a member function would have to be invoked.

Complex a, b, c;
// ...
a = b.add(c);

In C++, this expression tells the compiler to create three (3) objects on the stack, perform addition, and copy the resultant value from the temporary object into the existing object a.

However, in Java, operator= doesn't perform value copy for reference types, and users can only create new reference types, not value types. So for a user-defined type named Complex, assignment means to copy a reference to an existing value.

Consider instead:

b.set(1, 0); // initialize to real number '1'
a = b; 
b.set(2, 0);
assert( !a.equals(b) ); // this assertion will fail

In C++, this copies the value, so the comparison will result not-equal. In Java, operator= performs reference copy, so a and b are now referring to the same value. As a result, the comparison will produce 'equal', since the object will compare equal to itself.

The difference between copies and references only adds to the confusion of operator overloading. As @Sebastian mentioned, Java and C# both have to deal with value and reference equality separately -- operator+ would likely deal with values and objects, but operator= is already implemented to deal with references.

In C++, you should only be dealing with one kind of comparison at a time, so it can be less confusing. For example, on Complex, operator= and operator== are both working on values -- copying values and comparing values respectively.

Check if a string is a date value

_x000D_
_x000D_
document.getElementById('r1').innerHTML = dayjs('sdsdsd').isValid()_x000D_
_x000D_
document.getElementById('r2').innerHTML = dayjs('2/6/20').isValid()
_x000D_
<script src="https://unpkg.com/[email protected]/dayjs.min.js"></script>_x000D_
_x000D_
<p>'sdsdsd' is a date: <span id="r1"></span></p>_x000D_
<p>'2/6/20' is a date: <span id="r2"></span></p>
_x000D_
_x000D_
_x000D_

A light weight library is ready for you: Day.js

.datepicker('setdate') issues, in jQuery

If you would like to support really old browsers you should parse the date string, since using the ISO8601 date format with the Date constructor is not supported pre IE9:

var queryDate = '2009-11-01',
    dateParts = queryDate.match(/(\d+)/g)
    realDate = new Date(dateParts[0], dateParts[1] - 1, dateParts[2]);  
                                    // months are 0-based!
// For >= IE9
var realDate = new Date('2009-11-01');  

$('#datePicker').datepicker({ dateFormat: 'yy-mm-dd' }); // format to show
$('#datePicker').datepicker('setDate', realDate);

Check the above example here.

How are Anonymous inner classes used in Java?

Anonymous inner classes are effectively closures, so they can be used to emulate lambda expressions or "delegates". For example, take this interface:

public interface F<A, B> {
   B f(A a);
}

You can use this anonymously to create a first-class function in Java. Let's say you have the following method that returns the first number larger than i in the given list, or i if no number is larger:

public static int larger(final List<Integer> ns, final int i) {
  for (Integer n : ns)
     if (n > i)
        return n;
  return i;
}

And then you have another method that returns the first number smaller than i in the given list, or i if no number is smaller:

public static int smaller(final List<Integer> ns, final int i) {
   for (Integer n : ns)
      if (n < i)
         return n;
   return i;
}

These methods are almost identical. Using the first-class function type F, we can rewrite these into one method as follows:

public static <T> T firstMatch(final List<T> ts, final F<T, Boolean> f, T z) {
   for (T t : ts)
      if (f.f(t))
         return t;
   return z;
}

You can use an anonymous class to use the firstMatch method:

F<Integer, Boolean> greaterThanTen = new F<Integer, Boolean> {
   Boolean f(final Integer n) {
      return n > 10;
   }
};
int moreThanMyFingersCanCount = firstMatch(xs, greaterThanTen, x);

This is a really contrived example, but its easy to see that being able to pass functions around as if they were values is a pretty useful feature. See "Can Your Programming Language Do This" by Joel himself.

A nice library for programming Java in this style: Functional Java.

How to save an image to localStorage and display it on the next page?

I have come up with the same issue, instead of storing images, that eventually overflow the local storage, you can just store the path to the image. something like:

let imagen = ev.target.getAttribute('src');
arrayImagenes.push(imagen);

I want to show all tables that have specified column name

SELECT      T.TABLE_NAME, C.COLUMN_NAME
FROM        INFORMATION_SCHEMA.COLUMNS C
            INNER JOIN INFORMATION_SCHEMA.TABLES T ON T.TABLE_NAME = C.TABLE_NAME
WHERE       TABLE_TYPE = 'BASE TABLE'
            AND COLUMN_NAME = 'ColName'

This returns tables only and ignores views for anyone who is interested!

Load arrayList data into JTable

"The problem is that i cant find a way to set a fixed number of rows"

You don't need to set the number of rows. Use a TableModel. A DefaultTableModel in particular.

String col[] = {"Pos","Team","P", "W", "L", "D", "MP", "GF", "GA", "GD"};

DefaultTableModel tableModel = new DefaultTableModel(col, 0);
                                            // The 0 argument is number rows.

JTable table = new JTable(tableModel);

Then you can add rows to the tableModel with an Object[]

Object[] objs = {1, "Arsenal", 35, 11, 2, 2, 15, 30, 11, 19};

tableModel.addRow(objs);

You can loop to add your Object[] arrays.

Note: JTable does not currently allow instantiation with the input data as an ArrayList. It must be a Vector or an array.

See JTable and DefaultTableModel. Also, How to Use JTable tutorial

"I created an arrayList from it and I somehow can't find a way to store this information into a JTable."

You can do something like this to add the data

ArrayList<FootballClub> originalLeagueList = new ArrayList<FootballClub>();

originalLeagueList.add(new FootballClub(1, "Arsenal", 35, 11, 2, 2, 15, 30, 11, 19));
originalLeagueList.add(new FootballClub(2, "Liverpool", 30, 9, 3, 3, 15, 34, 18, 16));
originalLeagueList.add(new FootballClub(3, "Chelsea", 30, 9, 2, 2, 15, 30, 11, 19));
originalLeagueList.add(new FootballClub(4, "Man City", 29, 9, 2, 4, 15, 41, 15, 26));
originalLeagueList.add(new FootballClub(5, "Everton", 28, 7, 1, 7, 15, 23, 14, 9));
originalLeagueList.add(new FootballClub(6, "Tottenham", 27, 8, 4, 3, 15, 15, 16, -1));
originalLeagueList.add(new FootballClub(7, "Newcastle", 26, 8, 5, 2, 15, 20, 21, -1));
originalLeagueList.add(new FootballClub(8, "Southampton", 23, 6, 4, 5, 15, 19, 14, 5));

for (int i = 0; i < originalLeagueList.size(); i++){
   int position = originalLeagueList.get(i).getPosition();
   String name = originalLeagueList.get(i).getName();
   int points = originalLeagueList.get(i).getPoinst();
   int wins = originalLeagueList.get(i).getWins();
   int defeats = originalLeagueList.get(i).getDefeats();
   int draws = originalLeagueList.get(i).getDraws();
   int totalMatches = originalLeagueList.get(i).getTotalMathces();
   int goalF = originalLeagueList.get(i).getGoalF();
   int goalA = originalLeagueList.get(i).getGoalA();
   in ttgoalD = originalLeagueList.get(i).getTtgoalD();

   Object[] data = {position, name, points, wins, defeats, draws, 
                               totalMatches, goalF, goalA, ttgoalD};

   tableModel.add(data);

}

How do I get a python program to do nothing?

you can use pass inside if statement.

How to merge two PDF files into one in Java?

package article14;

import java.io.File;
import org.apache.pdfbox.pdmodel.PDDocument;
import org.apache.pdfbox.pdmodel.PDPage;
import org.apache.pdfbox.util.PDFMergerUtility;

public class Pdf
{
    public static void main(String args[])
    {
        new Pdf().createNew();
        new Pdf().combine();
        }

    public void combine()
    {
        try
        {
        PDFMergerUtility mergePdf = new PDFMergerUtility();
        String folder ="pdf";
        File _folder = new File(folder);
        File[] filesInFolder;
        filesInFolder = _folder.listFiles();
        for (File string : filesInFolder)
        {
            mergePdf.addSource(string);    
        }
    mergePdf.setDestinationFileName("Combined.pdf");
    mergePdf.mergeDocuments();
        }
        catch(Exception e)
        {

        }  
    }

public void createNew()
{
    PDDocument document = null;
    try
    {
        String filename="test.pdf";
        document=new PDDocument();
        PDPage blankPage = new PDPage();
        document.addPage( blankPage );
        document.save( filename );
    }
    catch(Exception e)
    {

    }
}

}

How to make a <button> in Bootstrap look like a normal link in nav-tabs?

Just make regular link look like button :)

<a href="#" role="button" class="btn btn-success btn-large">Click here!</a>

"role" inside a href code makes it look like button, ofc you can add more variables such as class.

HttpClient.GetAsync(...) never returns when using await/async

I'm going to put this in here more for completeness than direct relevance to the OP. I spent nearly a day debugging an HttpClient request, wondering why I was never getting back a response.

Finally found that I had forgotten to await the async call further down the call stack.

Feels about as good as missing a semicolon.

How to remove underline from a name on hover

_x000D_
_x000D_
legend.green-color{_x000D_
    color:green !important;_x000D_
}
_x000D_
_x000D_
_x000D_

Reduce left and right margins in matplotlib plot

inspired by Sammys answer above:

margins = {  #     vvv margin in inches
    "left"   :     1.5 / figsize[0],
    "bottom" :     0.8 / figsize[1],
    "right"  : 1 - 0.3 / figsize[0],
    "top"    : 1 - 1   / figsize[1]
}
fig.subplots_adjust(**margins)

Where figsize is the tuple that you used in fig = pyplot.figure(figsize=...)

Index (zero based) must be greater than or equal to zero

Change this line:

The 2 should be 0. Every count starts at 0.

//Aboutme.Text = String.Format("{2}", reader.GetString(0));//wrong

//Aboutme.Text = String.Format("{0}", reader.GetString(0));//correct

OpenCV !_src.empty() in function 'cvtColor' error

Check whether its the jpg, png, bmp file that you are providing and write the extension accordingly.

How to check existence of user-define table type in SQL Server 2008?

IF EXISTS(SELECT 1 FROM sys.types WHERE name = 'Person' AND is_table_type = 1 AND SCHEMA_ID('VAB') = schema_id)
DROP TYPE VAB.Person;
go
CREATE TYPE VAB.Person AS TABLE
(    PersonID               INT
    ,FirstName              VARCHAR(255)
    ,MiddleName             VARCHAR(255)
    ,LastName               VARCHAR(255)
    ,PreferredName          VARCHAR(255)
);

PHP Session Destroy on Log Out Button

The folder being password protected has nothing to do with PHP!

The method being used is called "Basic Authentication". There are no cross-browser ways to "logout" from it, except to ask the user to close and then open their browser...

Here's how you you could do it in PHP instead (fully remove your Apache basic auth in .htaccess or wherever it is first):

login.php:

<?php
session_start();
//change 'valid_username' and 'valid_password' to your desired "correct" username and password
if (! empty($_POST) && $_POST['user'] === 'valid_username' && $_POST['pass'] === 'valid_password')
{
    $_SESSION['logged_in'] = true;
    header('Location: /index.php');
}
else
{
    ?>

    <form method="POST">
    Username: <input name="user" type="text"><br>
    Password: <input name="pass" type="text"><br><br>
    <input type="submit" value="submit">
    </form>

    <?php
}

index.php

<?php
session_start();
if (! empty($_SESSION['logged_in']))
{
    ?>

    <p>here is my super-secret content</p>
    <a href='logout.php'>Click here to log out</a>

    <?php
}
else
{
    echo 'You are not logged in. <a href="login.php">Click here</a> to log in.';
}

logout.php:

<?php
session_start();
session_destroy();
echo 'You have been logged out. <a href="/">Go back</a>';

Obviously this is a very basic implementation. You'd expect the usernames and passwords to be in a database, not as a hardcoded comparison. I'm just trying to give you an idea of how to do the session thing.

Hope this helps you understand what's going on.

How to initialize a two-dimensional array in Python?

This is the best I've found for teaching new programmers, and without using additional libraries. I'd like something better though.

def initialize_twodlist(value):
    list=[]
    for row in range(10):
        list.append([value]*10)
    return list

How to read file from res/raw by name

Here are two approaches you can read raw resources using Kotlin.

You can get it by getting the resource id. Or, you can use string identifier in which you can programmatically change the filename with incrementation.

Cheers mate

// R.raw.data_post

this.context.resources.openRawResource(R.raw.data_post)
this.context.resources.getIdentifier("data_post", "raw", this.context.packageName)

How to disable scientific notation?

You can effectively remove scientific notation in printing with this code:

options(scipen=999)

How to convert a normal Git repository to a bare one?

Here is the definition of a bare repository from gitglossary:

A bare repository is normally an appropriately named directory with a .git suffix that does not have a locally checked-out copy of any of the files under revision control. That is, all of the Git administrative and control files that would normally be present in the hidden .git sub-directory are directly present in the repository.git directory instead, and no other files are present and checked out. Usually publishers of public repositories make bare repositories available.

I arrived here because I was playing around with a "local repository" and wanted to be able to do whatever I wanted as if it were a remote repository. I was just playing around, trying to learn about git. I'll assume that this is the situation for whoever wants to read this answer.

I would love for an expert opinion or some specific counter-examples, however it seems that (after rummaging through some git source code that I found) simply going to the file .git/config and setting the core attribute bare to true, git will let you do whatever you want to do to the repository remotely. I.e. the following lines should exist in .git/config:

[core]
    ...
    bare = true
...

(This is roughly what the command git config --bool core.bare true will do, which is probably recommended to deal with more complicated situations)

My justification for this claim is that, in the git source code, there seems to be two different ways of testing if a repo is bare or not. One is by checking a global variable is_bare_repository_cfg. This is set during some setup phase of execution, and reflects the value found in the .git/config file. The other is a function is_bare_repository(). Here is the definition of this function:

int is_bare_repository(void)
{
    /* if core.bare is not 'false', let's see if there is a work tree */
    return is_bare_repository_cfg && !get_git_work_tree();
} 

I've not the time nor expertise to say this with absolute confidence, but as far as I could tell if you have the bare attribute set to true in .git/config, this should always return 1. The rest of the function probably is for the following situation:

  1. core.bare is undefined (i.e. neither true nor false)
  2. There is no worktree (i.e. the .git subdirectory is the main directory)

I'll experiment with it when I can later, but this would seem to indicate that setting core.bare = true is equivalent to removeing core.bare from the config file and setting up the directories properly.

At any rate, setting core.bare = true certainly will let you push to it, but I'm not sure if the presence of project files will cause some other operations to go awry. It's interesting and I suppose instructive to push to the repository and see what happened locally (i.e. run git status and make sense of the results).

How I can delete in VIM all text from current line to end of file?

dG will delete from the current line to the end of file

dCtrl+End will delete from the cursor to the end of the file

But if this file is as large as you say, you may be better off reading the first few lines with head rather than editing and saving the file.

head hugefile > firstlines

(If you are on Windows you can use the Win32 port of head)

Clearing a string buffer/builder after loop

public void clear(StringBuilder s) {
    s.setLength(0);
}

Usage:

StringBuilder v = new StringBuilder();
clear(v);

for readability, I think this is the best solution.

How to import component into another root component in Angular 2

above answers In simple words, you have to register under @NgModule's

declarations: [
    AppComponent, YourNewComponentHere
  ] 

of app.module.ts

do not forget to import that component.

SQL SERVER: Check if variable is null and then assign statement for Where Clause

Try the following:

if ((select VisitCount from PageImage where PID=@pid and PageNumber=5) is NULL)
begin
    update PageImage
    set VisitCount=1
    where PID=@pid and PageNumber=@pageno
end 
else
begin
    update PageImage 
    set VisitCount=VisitCount+1
    where PID=@pid and PageNumber=@pageno
end

How can you search Google Programmatically Java API

To search google using API you should use Google Custom Search, scraping web page is not allowed

In java you can use CustomSearch API Client Library for Java

The maven dependency is:

<dependency>
    <groupId>com.google.apis</groupId>
    <artifactId>google-api-services-customsearch</artifactId>
    <version>v1-rev57-1.23.0</version>
</dependency> 

Example code searching using Google CustomSearch API Client Library

public static void main(String[] args) throws GeneralSecurityException, IOException {

    String searchQuery = "test"; //The query to search
    String cx = "002845322276752338984:vxqzfa86nqc"; //Your search engine

    //Instance Customsearch
    Customsearch cs = new Customsearch.Builder(GoogleNetHttpTransport.newTrustedTransport(), JacksonFactory.getDefaultInstance(), null) 
                   .setApplicationName("MyApplication") 
                   .setGoogleClientRequestInitializer(new CustomsearchRequestInitializer("your api key")) 
                   .build();

    //Set search parameter
    Customsearch.Cse.List list = cs.cse().list(searchQuery).setCx(cx); 

    //Execute search
    Search result = list.execute();
    if (result.getItems()!=null){
        for (Result ri : result.getItems()) {
            //Get title, link, body etc. from search
            System.out.println(ri.getTitle() + ", " + ri.getLink());
        }
    }

}

As you can see you will need to request an api key and setup an own search engine id, cx.

Note that you can search the whole web by selecting "Search entire web" on basic tab settings during setup of cx, but results will not be exactly the same as a normal browser google search.

Currently (date of answer) you get 100 api calls per day for free, then google like to share your profit.

Where can I find the error logs of nginx, using FastCGI and Django?

It is a good practice to set where the access log should be in nginx configuring file . Using acces_log /path/ Like this.

keyval $remote_addr:$http_user_agent $seen zone=clients;

server { listen 443 ssl;

ssl_protocols TLSv1 TLSv1.1 TLSv1.2;
ssl_ciphers   HIGH:!aNULL:!MD5;

if ($seen = "") {
    set $seen  1;
    set $logme 1;
}
access_log  /tmp/sslparams.log sslparams if=$logme;
error_log  /pathtolog/error.log;
# ...
}

Documentation for using JavaScript code inside a PDF file

The comprehensive place for Acrobat JavaScript documentation is the Acrobat SDK, which can be downloaded from the Adobe website. In the Documentation section, you will find all the material needed to work with Acrobat JavaScript.

To complete the documentation you may in addition get the specification of the JavaScript Core. My book of choice for that is "JavaScript, the Definitive Guide" by David Flanagan, published by O'Reilly.

how to set font size based on container size?

It cannot be accomplished with css font-size

Assuming that "external factors" you are referring to could be picked up by media queries, you could use them - adjustments will likely have to be limited to a set of predefined sizes.

jQuery UI DatePicker to show month year only

I know it's a little late response, but I got the same problem a couple of days before and I have came with a nice & smooth solution. First I found this great date picker here

Then I've just updated the CSS class (jquery.calendarPicker.css) that comes with the example like this:

.calMonth {
  /*border-bottom: 1px dashed #666;
  padding-bottom: 5px;
  margin-bottom: 5px;*/
}

.calDay 
{
    display:none;
}

The plugin fires an event DateChanged when you change anything, so it doesn't matter that you are not clicking on a day (and it fits nice as a year and month picker)

Hope it helps!

How do you clear Apache Maven's cache?

Delete the artifacts (or the full local repo) from c:\Users\<username>\.m2\repository by hand.

How do I get the computer name in .NET

2 more helpful methods: System.Environment.GetEnvironmentVariable("ComputerName" )

System.Environment.GetEnvironmentVariable("ClientName" ) to get the name of the user's PC if they're connected via Citrix XenApp or Terminal Services (aka RDS, RDP, Remote Desktop)

How to add Action bar options menu in Android Fragments

I am late for the answer but I think this is another solution which is not mentioned here so posting.

Step 1: Make a xml of menu which you want to add like I have to add a filter action on my action bar so I have created a xml filter.xml. The main line to notice is android:orderInCategory this will show the action icon at first or last wherever you want to show. One more thing to note down is the value, if the value is less then it will show at first and if value is greater then it will show at last.

filter.xml

<menu xmlns:android="http://schemas.android.com/apk/res/android"
    xmlns:app="http://schemas.android.com/apk/res-auto"
    xmlns:tools="http://schemas.android.com/tools" >


    <item
        android:id="@+id/action_filter"
        android:title="@string/filter"
        android:orderInCategory="10"
        android:icon="@drawable/filter"
        app:showAsAction="ifRoom" />


</menu>

Step 2: In onCreate() method of fragment just put the below line as mentioned, which is responsible for calling back onCreateOptionsMenu(Menu menu, MenuInflater inflater) method just like in an Activity.

@Override
    public void onCreate(Bundle savedInstanceState) {
        super.onCreate(savedInstanceState);
        setHasOptionsMenu(true);
    }

Step 3: Now add the method onCreateOptionsMenu which will be override as:

@Override
    public void onCreateOptionsMenu(Menu menu, MenuInflater inflater) {
        inflater.inflate(R.menu.filter, menu);  // Use filter.xml from step 1
    }

Step 4: Now add onOptionsItemSelected method by which you can implement logic whatever you want to do when you select the added action icon from actionBar:

@Override
    public boolean onOptionsItemSelected(MenuItem item) {
        int id = item.getItemId();
        if(id == R.id.action_filter){
            //Do whatever you want to do 
            return true;
        }

        return super.onOptionsItemSelected(item);
    }

How to import a module in Python with importlib.import_module

And don't forget to create a __init__.py with each folder/subfolder (even if they are empty)

Convert String to Carbon

Try this

$date = Carbon::parse(date_format($youttimestring,'d/m/Y H:i:s'));
echo $date;

How to convert number of minutes to hh:mm format in TSQL?

For those who need convert minutes to time with more than 24h format:

DECLARE @minutes int = 7830
SELECT CAST(@minutes / 60 AS VARCHAR(8)) + ':' + FORMAT(@minutes % 60, 'D2') AS [Time]

Result:

130:30

Commit history on remote repository

A fast way of doing this is to clone using the --bare keyword and then check the log:

git clone --bare git@giturl tmpdir
cd tmpdir
git log branch

Eclipse returns error message "Java was started but returned exit code = 1"

Directly changing eclipse file is not a good idea, no matter facet or ini, unless it could be changed in eclipse. Had the same problem, with jdk1.8 installed. Change it to jdk 1.7.enter image description here

Besides, according to https://wiki.eclipse.org/Eclipse/Installation, both LUNA and MARS need 1.7. So just ensure you have it installed.

How do I convert an integer to string as part of a PostgreSQL query?

You can cast an integer to a string in this way

intval::text

and so in your case

SELECT * FROM table WHERE <some integer>::text = 'string of numbers'

How to identify if a webpage is being loaded inside an iframe or directly into the browser window?

Use this javascript function as an example on how to accomplish this.

function isNoIframeOrIframeInMyHost() {
// Validation: it must be loaded as the top page, or if it is loaded in an iframe 
// then it must be embedded in my own domain.
// Info: IF top.location.href is not accessible THEN it is embedded in an iframe 
// and the domains are different.
var myresult = true;
try {
    var tophref = top.location.href;
    var tophostname = top.location.hostname.toString();
    var myhref = location.href;
    if (tophref === myhref) {
        myresult = true;
    } else if (tophostname !== "www.yourdomain.com") {
        myresult = false;
    }
} catch (error) { 
  // error is a permission error that top.location.href is not accessible 
  // (which means parent domain <> iframe domain)!
    myresult = false;
}
return myresult;
}

Reference jars inside a jar

in eclipse, right click project, select RunAs -> Run Configuration and save your run configuration, this will be used when you next export as Runnable JARs

How to calculate UILabel height dynamically?

To make UILabel fit the dynamic content you can use lines property in property inspector and set that as 0.

And you don't require to do any coding for this.

For more details you can check below demonstration video

https://www.youtube.com/watch?v=rVeDS_OGslU

NameError: global name 'unicode' is not defined - in Python 3

Hope you are using Python 3 , Str are unicode by default, so please Replace Unicode function with String Str function.

if isinstance(unicode_or_str, str):    ##Replaces with str
    text = unicode_or_str
    decoded = False

How to go up a level in the src path of a URL in HTML?

Use .. to indicate the parent directory:

background-image: url('../images/bg.png');

How to escape comma and double quote at same time for CSV file?

Thanks to both Tony and Paul for the quick feedback, its very helpful. I actually figure out a solution through POJO. Here it is:

if (cell_value.indexOf("\"") != -1 || cell_value.indexOf(",") != -1) {
    cell_value = cell_value.replaceAll("\"", "\"\"");
    row.append("\"");
    row.append(cell_value);
    row.append("\"");
} else {
    row.append(cell_value);
}

in short if there is special character like comma or double quote within the string in side the cell, then first escape the double quote("\"") by adding additional double quote (like "\"\""), then put the whole thing into a double quote (like "\""+theWholeThing+"\"" )

How to sort an array of associative arrays by value of a given key in PHP?

This function is re-usable:

function usortarr(&$array, $key, $callback = 'strnatcasecmp') {
    uasort($array, function($a, $b) use($key, $callback) {
        return call_user_func($callback, $a[$key], $b[$key]);
    });
}

It works well on string values by default, but you'll have to sub the callback for a number comparison function if all your values are numbers.

Detecting a mobile browser

I have faced some scenarios where above answers dint work for me. So i came up with this. Might be helpful to someone.

if(/iPhone|iPad|iPod|Android|webOS|BlackBerry|Windows Phone/i.test(navigator.userAgent)
 || screen.availWidth < 480){
//code for mobile
}

It depends on your use case. If you focus on screen use screen.availWidth, or you can use document.body.clientWidth if you want to render based on document.

How does the @property decorator work in Python?

This following:

class C(object):
    def __init__(self):
        self._x = None

    @property
    def x(self):
        """I'm the 'x' property."""
        return self._x

    @x.setter
    def x(self, value):
        self._x = value

    @x.deleter
    def x(self):
        del self._x

Is the same as:

class C(object):
    def __init__(self):
        self._x = None

    def _x_get(self):
        return self._x

    def _x_set(self, value):
        self._x = value

    def _x_del(self):
        del self._x

    x = property(_x_get, _x_set, _x_del, 
                    "I'm the 'x' property.")

Is the same as:

class C(object):
    def __init__(self):
        self._x = None

    def _x_get(self):
        return self._x

    def _x_set(self, value):
        self._x = value

    def _x_del(self):
        del self._x

    x = property(_x_get, doc="I'm the 'x' property.")
    x = x.setter(_x_set)
    x = x.deleter(_x_del)

Is the same as:

class C(object):
    def __init__(self):
        self._x = None

    def _x_get(self):
        return self._x
    x = property(_x_get, doc="I'm the 'x' property.")

    def _x_set(self, value):
        self._x = value
    x = x.setter(_x_set)

    def _x_del(self):
        del self._x
    x = x.deleter(_x_del)

Which is the same as :

class C(object):
    def __init__(self):
        self._x = None

    @property
    def x(self):
        """I'm the 'x' property."""
        return self._x

    @x.setter
    def x(self, value):
        self._x = value

    @x.deleter
    def x(self):
        del self._x

Get day of week using NSDate

Swift 3 : Xcode 8 helper function:

func getDayOfWeek(fromDate date: Date) -> String? {
    let cal = Calendar(identifier: .gregorian)
    let dayOfWeek = cal.component(.weekday, from: date)
    switch dayOfWeek {
     case 1:
        return "Sunday"
    case 2:
        return "Monday"
    case 3:
        return "Tuesday"
    case 4:
        return "Wednesday"
    case 5:
        return "Thursday"
    case 6:
        return "Friday"
    case 7:
        return "Saturday"
    default:
        return nil
    }
}

Perl read line by line

In bash foo is the name of the variable, and $ is an operator which means 'get the value of'.

In perl $foo is the name of the variable.

Getting the name of a variable as a string

I think it's so difficult to do this in Python because of the simple fact that you never will not know the name of the variable you're using. So, in his example, you could do:

Instead of:

list_of_dicts = [n_jobs, users, queues, priorities]

dict_of_dicts = {"n_jobs" : n_jobs, "users" : users, "queues" : queues, "priorities" : priorities}

How to parse string into date?

CONVERT(datetime, '24.04.2012', 104)

Should do the trick. See here for more info: CAST and CONVERT (Transact-SQL)

perform an action on checkbox checked or unchecked event on html form

<form>
    syn<input type="checkbox" name="checkfield" id="g01-01" />
</form>

js:

$('#g01-01').on('change',function(){
    var _val = $(this).is(':checked') ? 'checked' : 'unchecked';
    alert(_val);
});

matplotlib.pyplot will not forget previous plots - how can I flush/refresh?

I discovered that this behaviour only occurs after running a particular script, similar to the one in the question. I have no idea why it occurs.

It works (refreshes the graphs) if I put

plt.clf()
plt.cla()
plt.close()

after every plt.show()

How to print all session variables currently set?

echo '<pre>';
var_dump($_SESSION);
echo '</pre>';

Or you can use print_r if you don't care about types. If you use print_r, you can make the second argument TRUE so it will return instead of echo, useful for...

echo '<pre>' . print_r($_SESSION, TRUE) . '</pre>';

How to use SharedPreferences in Android to store, fetch and edit values

To store values in shared preferences:

SharedPreferences sp = PreferenceManager.getDefaultSharedPreferences(this);
SharedPreferences.Editor editor = sp.edit();
editor.putString("Name","Jayesh");
editor.commit();

To retrieve values from shared preferences:

SharedPreferences sp = PreferenceManager.getDefaultSharedPreferences(this);
String name = sp.getString("Name", ""); // Second parameter is the default value.

How to delete last character in a string in C#?

string source;
// source gets initialized
string dest;
if (source.Length > 0)
{
    dest = source.Substring(0, source.Length - 1);
}

How do I get a Date without time in Java?

The standard answer to these questions is to use Joda Time. The API is better and if you're using the formatters and parsers you can avoid the non-intuitive lack of thread safety of SimpleDateFormat.

Using Joda means you can simply do:

LocalDate d = new LocalDate();

Update:: Using java 8 this can be acheived using

LocalDate date = LocalDate.now();

How to fix "set SameSite cookie to none" warning?

I ended up fixing our Ubuntu 18.04 / Apache 2.4.29 / PHP 7.2 install for Chrome 80 by installing mod_headers:

a2enmod headers

Adding the following directive to our Apache VirtualHost configurations:

Header edit Set-Cookie ^(.*)$ "$1; Secure; SameSite=None"

And restarting Apache:

service apache2 restart

In reviewing the docs (http://www.balkangreenfoundation.org/manual/en/mod/mod_headers.html) I noticed the "always" condition has certain situations where it does not work from the same pool of response headers. Thus not using "always" is what worked for me with PHP but the docs suggest that if you want to cover all your bases you could add the directive both with and without "always". I have not tested that.

How to create Password Field in Model Django

See my code which may help you. models.py

from django.db import models

class Customer(models.Model):
    name = models.CharField(max_length=100)
    email = models.EmailField(max_length=100)
    password = models.CharField(max_length=100)
    instrument_purchase = models.CharField(max_length=100)
    house_no = models.CharField(max_length=100)
    address_line1 = models.CharField(max_length=100)
    address_line2 = models.CharField(max_length=100)
    telephone = models.CharField(max_length=100)
    zip_code = models.CharField(max_length=20)
    state = models.CharField(max_length=100)
    country = models.CharField(max_length=100)

    def __str__(self):
        return self.name

forms.py

from django import forms
from models import *

class CustomerForm(forms.ModelForm):
    password = forms.CharField(widget=forms.PasswordInput)

    class Meta:
        model = Customer
        fields = ('name', 'email', 'password', 'instrument_purchase', 'house_no', 'address_line1', 'address_line2', 'telephone', 'zip_code', 'state', 'country')

How to set all elements of an array to zero or any same value?

You could use memset, if you sure about the length.

memset(ptr, 0x00, length)

Remove specific characters from a string in Python

Using filter, you'd just need one line

line = filter(lambda char: char not in " ?.!/;:", line)

This treats the string as an iterable and checks every character if the lambda returns True:

>>> help(filter)
Help on built-in function filter in module __builtin__:

filter(...)
    filter(function or None, sequence) -> list, tuple, or string

    Return those items of sequence for which function(item) is true.  If
    function is None, return the items that are true.  If sequence is a tuple
    or string, return the same type, else return a list.

Passing parameter to controller action from a Html.ActionLink

Addition to the accepted answer:

if you are going to use

 @Html.ActionLink("LinkName", "ActionName", "ControllerName", new { @id = idValue, @secondParam= = 2 },null)

this will create actionlink where you can't create new custom attribute or style for the link.

However, the 4th parameter in ActionLink extension will solve that problem. Use the 4th parameter for customization in your way.

 @Html.ActionLink("LinkName", "ActionName", "ControllerName", new { @id = idValue, @secondParam= = 2 }, new { @class = "btn btn-info", @target = "_blank" })

Where does application data file actually stored on android device?

On Android 4.4 KitKat, I found mine in: /sdcard/Android/data/<app.package.name>

Setting background colour of Android layout element

If you want to change a color quickly (and you don't have Hex numbers memorized) android has a few preset colors you can access like this:

android:background="@android:color/black"

There are 15 colors you can choose from which is nice for testing things out quickly, and you don't need to set up additional files.

Setting up a values/colors.xml file and using straight Hex like explained above will still work.

Find first element in a sequence that matches a predicate

I don't think there's anything wrong with either solutions you proposed in your question.

In my own code, I would implement it like this though:

(x for x in seq if predicate(x)).next()

The syntax with () creates a generator, which is more efficient than generating all the list at once with [].

css with background image without repeating the image

body {
    background: url(images/image_name.jpg) no-repeat center center fixed; 
    -webkit-background-size: cover;
    -moz-background-size: cover;
    -o-background-size: cover;
    background-size: cover;
}

Here is a good solution to get your image to cover the full area of the web app perfectly

KnockoutJs v2.3.0 : Error You cannot apply bindings multiple times to the same element

If you are reusing an element over and over (A bootstrap modal dialog in my case), then calling ko.applyBindings(el) multiple times will cause this problem.

Instead just do it once like this:

if (!applied) {
    ko.applyBindings(el);
    applied = true;
}

Or like this:

var apply = function (viewModel, containerElement) {
    ko.applyBindings(viewModel, containerElement);
    apply = function() {}; // only allow this function to be called once.
}

PS: This might happen more often to you if you use the mapping plugin and convert your JSON data to observables.

SSIS Connection Manager Not Storing SQL Password

There is easy way of doing this. I don't know why people are giving complicated answers.

Double click SSIS package. Then go to connection manager, select DestinationConnectionOLDB and then add password next to login field.

Example: Data Source=SysproDB1;User ID=test;password=test;Initial Catalog=ASBuiltDW;Provider=SQLNCLI11;Auto Translate=false;

Do same for SourceConnectionOLDB.

Allow multiple roles to access controller action

Another clear solution, you can use constants to keep convention and add multiple [Authorize] attributes. Check this out:

public static class RolesConvention
{
    public const string Administrator = "Administrator";
    public const string Guest = "Guest";
}

Then in the controller:

[Authorize(Roles = RolesConvention.Administrator )]
[Authorize(Roles = RolesConvention.Guest)]
[Produces("application/json")]
[Route("api/[controller]")]
public class MyController : Controller

Remove HTML tags from a String

You might want to replace <br/> and </p> tags with newlines before stripping the HTML to prevent it becoming an illegible mess as Tim suggests.

The only way I can think of removing HTML tags but leaving non-HTML between angle brackets would be check against a list of HTML tags. Something along these lines...

replaceAll("\\<[\s]*tag[^>]*>","")

Then HTML-decode special characters such as &amp;. The result should not be considered to be sanitized.

Array.push() and unique items

Yep, it's a small mistake.

if(this.items.indexOf(item) === -1) {
    this.items.push(item);
    console.log(this.items);
}

How to restrict SSH users to a predefined set of commands after login?

Another way of looking at this is using POSIX ACLs, it needs to be supported by your file system, however you can have fine-grained tuning of all commands in linux the same way you have the same control on Windows (just without the nicer UI). link

Another thing to look into is PolicyKit.

You'll have to do quite a bit of googling to get everything working as this is definitely not a strength of Linux at the moment.

Accessing member of base class

Working example. Notes below.

class Animal {
    constructor(public name) {
    }

    move(meters) {
        alert(this.name + " moved " + meters + "m.");
    }
}

class Snake extends Animal {
    move() {
        alert(this.name + " is Slithering...");
        super.move(5);
    }
}

class Horse extends Animal {
    move() {
        alert(this.name + " is Galloping...");
        super.move(45);
    }
}

var sam = new Snake("Sammy the Python");
var tom: Animal = new Horse("Tommy the Palomino");

sam.move();
tom.move(34);
  1. You don't need to manually assign the name to a public variable. Using public name in the constructor definition does this for you.

  2. You don't need to call super(name) from the specialised classes.

  3. Using this.name works.

Notes on use of super.

This is covered in more detail in section 4.9.2 of the language specification.

The behaviour of the classes inheriting from Animal is not dissimilar to the behaviour in other languages. You need to specify the super keyword in order to avoid confusion between a specialised function and the base class function. For example, if you called move() or this.move() you would be dealing with the specialised Snake or Horse function, so using super.move() explicitly calls the base class function.

There is no confusion of properties, as they are the properties of the instance. There is no difference between super.name and this.name - there is simply this.name. Otherwise you could create a Horse that had different names depending on whether you were in the specialized class or the base class.

How to delete or add column in SQLITE?

As SQLite has limited support to ALTER TABLE so you can only ADD column at end of the table OR CHANGE TABLE_NAME in SQLite.

Here is the Best Answer of HOW TO DELETE COLUMN FROM SQLITE?

visit Delete column from SQLite table

How do I improve ASP.NET MVC application performance?

The basic suggestion is to follow REST principles and the following points ties some of these principals to the ASP.NET MVC framework:

  1. Make your controllers stateless - this is more of a 'Web performance / scalability' suggestion (as opposed to micro/machine level performance) and a major design decision that would affect your applications future - especially in case it becomes popular or if you need some fault tolerance for example.
    • Do not use Sessions
    • Do not use tempdata - which uses sessions
    • Do not try to 'cache' everything 'prematurely'.
  2. Use Forms Authentication
    • Keep your frequently accessed sensitive data in the authentication ticket
  3. Use cookies for frequently accessed non sensitive information
  4. Make your resources cachable on the web
  5. Compile your JavaScript. There is Closure compiler library to do it as well (sure there are others, just search for 'JavaScript compiler' too)
  6. Use CDNs (Content Delivery Network) - especially for your large media files and so on.
  7. Consider different types of storage for your data, for example, files, key/value stores, etc. - not only SQL Server
  8. Last but not least, test your web site for performance

Failed to install *.apk on device 'emulator-5554': EOF

Wipe Data and restart the virtual device again fix the issue in my case.

enter image description here

Algorithm to calculate the number of divisors of a given number

The sieve of Atkin is an optimized version of the sieve of Eratosthenes which gives all prime numbers up to a given integer. You should be able to google this for more detail.

Once you have that list, it's a simple matter to divide your number by each prime to see if it's an exact divisor (i.e., remainder is zero).

The basic steps calculating the divisors for a number (n) are [this is pseudocode converted from real code so I hope I haven't introduced errors]:

for z in 1..n:
    prime[z] = false
prime[2] = true;
prime[3] = true;

for x in 1..sqrt(n):
    xx = x * x

    for y in 1..sqrt(n):
        yy = y * y

        z = 4*xx+yy
        if (z <= n) and ((z mod 12 == 1) or (z mod 12 == 5)):
            prime[z] = not prime[z]

        z = z-xx
        if (z <= n) and (z mod 12 == 7):
            prime[z] = not prime[z]

        z = z-yy-yy
        if (z <= n) and (x > y) and (z mod 12 == 11):
            prime[z] = not prime[z]

for z in 5..sqrt(n):
    if prime[z]:
        zz = z*z
        x = zz
        while x <= limit:
            prime[x] = false
            x = x + zz

for z in 2,3,5..n:
    if prime[z]:
        if n modulo z == 0 then print z

How to select all elements with a particular ID in jQuery?

Can you assign a unique CSS class to each distinct timer? That way you could use the selector for the CSS class, which would work fine with multiple div elements.

Match line break with regular expression

You could search for:

<li><a href="#">[^\n]+

And replace with:

$0</a>

Where $0 is the whole match. The exact semantics will depend on the language are you using though.


WARNING: You should avoid parsing HTML with regex. Here's why.

SVN- How to commit multiple files in a single shot

Use a changeset. You can add as many files as you like to the changeset, all at once, or over several commands; and then commit them all in one go.

Why would you use String.Equals over ==?

I want to add that there is another difference. It is related to what Andrew posts.

It is also related to a VERY annoying to find bug in our software. See the following simplified example (I also omitted the null check).

public const int SPECIAL_NUMBER = 213;

public bool IsSpecialNumberEntered(string numberTextBoxTextValue)
{
    return numberTextBoxTextValue.Equals(SPECIAL_NUMBER)
}

This will compile and always return false. While the following will give a compile error:

public const int SPECIAL_NUMBER = 213;

public bool IsSpecialNumberEntered(string numberTextBoxTextValue)
{
    return (numberTextBoxTextValue == SPECIAL_NUMBER);
}

We have had to solve a similar problem where someone compared enums of different type using Equals. You are going to read over this MANY times before realising it is the cause of the bug. Especially if the definition of SPECIAL_NUMBER is not near the problem area.

This is why I am really against the use of Equals in situations where is it not necessary. You lose a little bit of type-safety.

Jquery - How to make $.post() use contentType=application/json?

$.ajax({
  url:url,
  type:"POST",
  data:data,
  contentType:"application/json; charset=utf-8",
  dataType:"json",
  success: function(){
    ...
  }
})

See : jQuery.ajax()

How to get row from R data.frame

If you don't know the row number, but do know some values then you can use subset

x <- structure(list(A = c(5,    3.5, 3.25, 4.25,  1.5 ), 
                    B = c(4.25, 4,   4,    4.5,   4.5 ),
                    C = c(4.5,  2.5, 4,    2.25,  3   )
               ),
               .Names    = c("A", "B", "C"),
               class     = "data.frame",
               row.names = c(NA, -5L)
     )

subset(x, A ==5 & B==4.25 & C==4.5)

Using SED with wildcard

The asterisk (*) means "zero or more of the previous item".

If you want to match any single character use

sed -i 's/string-./string-0/g' file.txt

If you want to match any string (i.e. any single character zero or more times) use

sed -i 's/string-.*/string-0/g' file.txt

Index of duplicates items in a python list

Using new "Counter" class in collections module, based on lazyr's answer:

>>> import collections
>>> def duplicates(n): #n="123123123"
...     counter=collections.Counter(n) #{'1': 3, '3': 3, '2': 3}
...     dups=[i for i in counter if counter[i]!=1] #['1','3','2']
...     result={}
...     for item in dups:
...             result[item]=[i for i,j in enumerate(n) if j==item] 
...     return result
... 
>>> duplicates("123123123")
{'1': [0, 3, 6], '3': [2, 5, 8], '2': [1, 4, 7]}

<meta charset="utf-8"> vs <meta http-equiv="Content-Type">

Both forms of the meta charset declaration are equivalent and should work the same across browsers. But, there are a few things you need to remember when declaring your web files character-set as UTF-8:

  1. Save your file(s) in UTF-8 encoding without the byte-order mark (BOM).
  2. Declare the encoding in your HTML files using meta charset (like above).
  3. Your web server must serve your files, declaring the UTF-8 encoding in the Content-Type HTTP header.

Apache servers are configured to serve files in ISO-8859-1 by default, so you need to add the following line to your .htaccess file:

AddDefaultCharset UTF-8

This will configure Apache to serve your files declaring UTF-8 encoding in the Content-Type response header, but your files must be saved in UTF-8 (without BOM) to begin with.

Notepad cannot save your files in UTF-8 without the BOM. A free editor that can is Notepad++. On the program menu bar, select "Encoding > Encode in UTF-8 without BOM". You can also open files and re-save them in UTF-8 using "Encoding > Convert to UTF-8 without BOM".

More on the Byte Order Mark (BOM) at Wikipedia.

How can I use Bash syntax in Makefile targets?

There is a way to do this without explicitly setting your SHELL variable to point to bash. This can be useful if you have many makefiles since SHELL isn't inherited by subsequent makefiles or taken from the environment. You also need to be sure that anyone who compiles your code configures their system this way.

If you run sudo dpkg-reconfigure dash and answer 'no' to the prompt, your system will not use dash as the default shell. It will then point to bash (at least in Ubuntu). Note that using dash as your system shell is a bit more efficient though.

Subprocess check_output returned non-zero exit status 1

The word check_ in the name means that if the command (the shell in this case that returns the exit status of the last command (yum in this case)) returns non-zero status then it raises CalledProcessError exception. It is by design. If the command that you want to run may return non-zero status on success then either catch this exception or don't use check_ methods. You could use subprocess.call in your case because you are ignoring the captured output, e.g.:

import subprocess

rc = subprocess.call(['grep', 'pattern', 'file'],
                     stdout=subprocess.DEVNULL, stderr=subprocess.STDOUT)
if rc == 0: # found
   ...
elif rc == 1: # not found
   ...
elif rc > 1: # error
   ...

You don't need shell=True to run the commands from your question.

C#: HttpClient with POST parameters

As Ben said, you are POSTing your request ( HttpMethod.Post specified in your code )

The querystring (get) parameters included in your url probably will not do anything.

Try this:

string url = "http://myserver/method";    
string content = "param1=1&param2=2";
HttpClientHandler handler = new HttpClientHandler();
HttpClient httpClient = new HttpClient(handler);
HttpRequestMessage request = new HttpRequestMessage(HttpMethod.Post, url);
HttpResponseMessage response = await httpClient.SendAsync(request,content);

HTH,

bovako

Combine two data frames by rows (rbind) when they have different sets of columns

gtools/smartbind didnt like working with Dates, probably because it was as.vectoring. So here's my solution...

sbind = function(x, y, fill=NA) {
    sbind.fill = function(d, cols){ 
        for(c in cols)
            d[[c]] = fill
        d
    }

    x = sbind.fill(x, setdiff(names(y),names(x)))
    y = sbind.fill(y, setdiff(names(x),names(y)))

    rbind(x, y)
}

How to get the timezone offset in GMT(Like GMT+7:00) from android device?

public static String getCurrentTimezoneOffset() {

    TimeZone tz = TimeZone.getDefault();  
    Calendar cal = GregorianCalendar.getInstance(tz);
    int offsetInMillis = tz.getOffset(cal.getTimeInMillis());

    String offset = String.format("%02d:%02d", Math.abs(offsetInMillis / 3600000), Math.abs((offsetInMillis / 60000) % 60));
    offset = (offsetInMillis >= 0 ? "+" : "-") + offset;

    return offset;
}

jQuery Multiple ID selectors

If you give each of these instances a class you can use

$('.yourClass').upload()

JPA: JOIN in JPQL

Join on one-to-many relation in JPQL looks as follows:

select b.fname, b.lname from Users b JOIN b.groups c where c.groupName = :groupName 

When several properties are specified in select clause, result is returned as Object[]:

Object[] temp = (Object[]) em.createNamedQuery("...")
    .setParameter("groupName", groupName)
    .getSingleResult(); 
String fname = (String) temp[0];
String lname = (String) temp[1];

By the way, why your entities are named in plural form, it's confusing. If you want to have table names in plural, you may use @Table to specify the table name for the entity explicitly, so it doesn't interfere with reserved words:

@Entity @Table(name = "Users")     
public class User implements Serializable { ... } 

How do I store data in local storage using Angularjs?

You can use localStorage for the purpose.

Steps:

  1. add ngStorage.min.js in your file
  2. add ngStorage dependency in your module
  3. add $localStorage module in your controller
  4. use $localStorage.key = value

How to print third column to last column?

Perl solution:

perl -lane 'splice @F,0,2; print join " ",@F' file

These command-line options are used:

  • -n loop around every line of the input file, do not automatically print every line

  • -l removes newlines before processing, and adds them back in afterwards

  • -a autosplit mode – split input lines into the @F array. Defaults to splitting on whitespace

  • -e execute the perl code

splice @F,0,2 cleanly removes columns 0 and 1 from the @F array

join " ",@F joins the elements of the @F array, using a space in-between each element

If your input file is comma-delimited, rather than space-delimited, use -F, -lane


Python solution:

python -c "import sys;[sys.stdout.write(' '.join(line.split()[2:]) + '\n') for line in sys.stdin]" < file

Programmatically Creating UILabel

Swift 3:

let label = UILabel(frame: CGRect(x:0,y: 0,width: 250,height: 50))
label.textAlignment = .center
label.textColor = .white
label.font = UIFont(name: "Avenir-Light", size: 15.0)
label.text = "This is a Label"
self.view.addSubview(label)

Is there a java setting for disabling certificate validation?

On my Mac that I'm sure I'm not going to allow java anyplace other than a specific site, I was able to use Preferences->Java to bring up the Java control panel and turned the checking off. If DLink ever fixes their certificate, I'll turn it back on.

Java control panel - Advanced

Converting a Java Keystore into PEM Format

It's pretty straightforward, using jdk6 at least...

bash$ keytool -keystore foo.jks -genkeypair -alias foo \
        -dname 'CN=foo.example.com,L=Melbourne,ST=Victoria,C=AU'
Enter keystore password:  
Re-enter new password: 
Enter key password for 
        (RETURN if same as keystore password):  
bash$ keytool -keystore foo.jks -exportcert -alias foo | \
       openssl x509 -inform der -text
Enter keystore password:  asdasd
Certificate:
    Data:
        Version: 3 (0x2)
        Serial Number: 1237334757 (0x49c03ae5)
        Signature Algorithm: dsaWithSHA1
        Issuer: C=AU, ST=Victoria, L=Melbourne, CN=foo.example.com
        Validity
            Not Before: Mar 18 00:05:57 2009 GMT
            Not After : Jun 16 00:05:57 2009 GMT
        Subject: C=AU, ST=Victoria, L=Melbourne, CN=foo.example.com
        Subject Public Key Info:
            Public Key Algorithm: dsaEncryption
            DSA Public Key:
                pub: 
                    00:e2:66:5c:e0:2e:da:e0:6b:a6:aa:97:64:59:14:
                    7e:a6:2e:5a:45:f9:2f:b5:2d:f4:34:27:e6:53:c7:
 

bash$ keytool -importkeystore -srckeystore foo.jks \
       -destkeystore foo.p12 \
       -srcstoretype jks \
       -deststoretype pkcs12
Enter destination keystore password:  
Re-enter new password: 
Enter source keystore password:  
Entry for alias foo successfully imported.
Import command completed:  1 entries successfully imported, 0 entries failed or cancelled

bash$ openssl pkcs12 -in foo.p12 -out foo.pem
Enter Import Password:
MAC verified OK
Enter PEM pass phrase:
Verifying - Enter PEM pass phrase:

bash$ openssl x509 -text -in foo.pem
Certificate:
    Data:
        Version: 3 (0x2)
        Serial Number: 1237334757 (0x49c03ae5)
        Signature Algorithm: dsaWithSHA1
        Issuer: C=AU, ST=Victoria, L=Melbourne, CN=foo.example.com
        Validity
            Not Before: Mar 18 00:05:57 2009 GMT
            Not After : Jun 16 00:05:57 2009 GMT
        Subject: C=AU, ST=Victoria, L=Melbourne, CN=foo.example.com
        Subject Public Key Info:
            Public Key Algorithm: dsaEncryption
            DSA Public Key:
                pub: 
                    00:e2:66:5c:e0:2e:da:e0:6b:a6:aa:97:64:59:14:
                    7e:a6:2e:5a:45:f9:2f:b5:2d:f4:34:27:e6:53:c7:
 

bash$ openssl dsa -text -in foo.pem
read DSA key
Enter PEM pass phrase:
Private-Key: (1024 bit)
priv:
    00:8f:b1:af:55:63:92:7c:d2:0f:e6:f3:a2:f5:ff:
    1a:7a:fe:8c:39:dd
pub: 
    00:e2:66:5c:e0:2e:da:e0:6b:a6:aa:97:64:59:14:
    7e:a6:2e:5a:45:f9:2f:b5:2d:f4:34:27:e6:53:c7:



You end up with:

  • foo.jks - keystore in java format.
  • foo.p12 - keystore in PKCS#12 format.
  • foo.pem - all keys and certs from keystore, in PEM format.

(This last file can be split up into keys and certificates if you like.)


Command summary - to create JKS keystore:

keytool -keystore foo.jks -genkeypair -alias foo \
    -dname 'CN=foo.example.com,L=Melbourne,ST=Victoria,C=AU'

Command summary - to convert JKS keystore into PKCS#12 keystore, then into PEM file:

keytool -importkeystore -srckeystore foo.jks \
   -destkeystore foo.p12 \
   -srcstoretype jks \
   -deststoretype pkcs12

openssl pkcs12 -in foo.p12 -out foo.pem

if you have more than one certificate in your JKS keystore, and you want to only export the certificate and key associated with one of the aliases, you can use the following variation:

keytool -importkeystore -srckeystore foo.jks \
   -destkeystore foo.p12 \
   -srcalias foo \
   -srcstoretype jks \
   -deststoretype pkcs12

openssl pkcs12 -in foo.p12 -out foo.pem

Command summary - to compare JKS keystore to PEM file:

keytool -keystore foo.jks -exportcert -alias foo | \
   openssl x509 -inform der -text

openssl x509 -text -in foo.pem

openssl dsa -text -in foo.pem

How to Deep clone in javascript

Avoid use this method

let cloned = JSON.parse(JSON.stringify(objectToClone));

Why? this method will convert 'function,undefined' to null

const myObj = [undefined, null, function () {}, {}, '', true, false, 0, Symbol];

const IsDeepClone = JSON.parse(JSON.stringify(myObj));

console.log(IsDeepClone); //[null, null, null, {…}, "", true, false, 0, null]

try to use deepClone function.There are several above

Compiling an application for use in highly radioactive environments

I've really read a lot of great answers!

Here is my 2 cent: build a statistical model of the memory/register abnormality, by writing a software to check the memory or to perform frequent register comparisons. Further, create an emulator, in the style of a virtual machine where you can experiment with the issue. I guess if you vary junction size, clock frequency, vendor, casing, etc would observe a different behavior.

Even our desktop PC memory has a certain rate of failure, which however doesn't impair the day to day work.

How to print current date on python3?

I use this which is standard for every time

import datetime
now = datetime.datetime.now()
print ("Current date and time : ")
print (now.strftime("%Y-%m-%d %H:%M:%S"))

Add a UIView above all, even the navigation bar

I'd use a UIViewController subclass containing a "Container View" that embeds your others view controllers. You'll then be able to add the navigation controller inside the Container View (using the embed relationship segue for example).

See Implementing a Container View Controller

DateTime.MinValue and SqlDateTime overflow

Very simple avoid using DateTime.MinValue use System.Data.SqlTypes.SqlDateTime.MinValue instead.

How to create a DOM node as an object?

First make your template into a jQuery object:

 var template = $("<li><div class='bar'>bla</div></li>");

Then set the attributes and append it to the DOM.

 template.find('li').attr('id','1234');
 $(document.body).append(template);

Note that it however makes no sense at all to add a li directly to the DOM since li should always be children of ul or ol. Also it is better to not make jQuery parse raw HTML. Instead create a li, set its attributes. Create a div and set it's attributes. Insert the div into the li and then append the li to the DOM.

How can I check if a file exists in Perl?

if (-e $base_path)
{ 
 # code
}

-e is the 'existence' operator in Perl.

You can check permissions and other attributes using the code on this page.

How to write to error log file in PHP

We all know that PHP save errors in php_errors.log file.

But, that file contains a lot of data.

If we want to log our application data, we need to save it to a custom location.

We can use two parameters in the error_log function to achieve this.

http://php.net/manual/en/function.error-log.php

We can do it using:

error_log(print_r($v, TRUE), 3, '/var/tmp/errors.log');

Where,

print_r($v, TRUE) : logs $v (array/string/object) to log file. 3: Put log message to custom log file specified in the third parameter.

'/var/tmp/errors.log': Custom log file (This path is for Linux, we can specify other depending upon OS).

OR, you can use file_put_contents()

file_put_contents('/var/tmp/e.log', print_r($v, true), FILE_APPEND);

Where:

'/var/tmp/errors.log': Custom log file (This path is for Linux, we can specify other depending upon OS). print_r($v, TRUE) : logs $v (array/string/object) to log file. FILE_APPEND: Constant parameter specifying whether to append to the file if it exists, if file does not exist, new file will be created.

How to overwrite styling in Twitter Bootstrap

I know this is an old question but still, I came across a similar problem and i realized that my "not working" css code in my bootstrapOverload.css file was written after the media queries. when I moved it above media queries it started working.

Just in case someone else is facing the same problem

Detecting input change in jQuery?

// .blur is triggered when element loses focus

$('#target').blur(function() {
  alert($(this).val());
});

// To trigger manually use:

$('#target').blur();

how to convert .java file to a .class file

From the command line, run

javac ClassName.java

How to horizontally center an unordered list of unknown width?

The solution, if your list items can be display: inline is quite easy:

#footer { text-align: center; }
#footer ul { list-style: none; }
#footer ul li { display: inline; }

However, many times you must use display:block on your <li>s. The following CSS will work, in this case:

#footer { width: 100%; overflow: hidden; }
#footer ul { list-style: none; position: relative; float: left; display: block; left: 50%; }
#footer ul li { position: relative; float: left; display: block; right: 50%; }

How can I get a Dialog style activity window to fill the screen?

You may add this values to your style android:windowMinWidthMajor and android:windowMinWidthMinor

<style name="Theme_Dialog" parent="android:Theme.Holo.Dialog">
    ...
    <item name="android:windowMinWidthMajor">97%</item>
    <item name="android:windowMinWidthMinor">97%</item>
</style>

Convert NSArray to NSString in Objective-C

NSString * str = [componentsJoinedByString:@""];

and you have dic or multiple array then used bellow

NSString * result = [[array valueForKey:@"description"] componentsJoinedByString:@""];   

Declaring and initializing a string array in VB.NET

I believe you need to specify "Option Infer On" for this to work.

Option Infer allows the compiler to make a guess at what is being represented by your code, thus it will guess that {"stuff"} is an array of strings. With "Option Infer Off", {"stuff"} won't have any type assigned to it, ever, and so it will always fail, without a type specifier.

Option Infer is, I think On by default in new projects, but Off by default when you migrate from earlier frameworks up to 3.5.

Opinion incoming:

Also, you mention that you've got "Option Explicit Off". Please don't do this.

Setting "Option Explicit Off" means that you don't ever have to declare variables. This means that the following code will silently and invisibly create the variable "Y":

Dim X as Integer
Y = 3

This is horrible, mad, and wrong. It creates variables when you make typos. I keep hoping that they'll remove it from the language.

moving committed (but not pushed) changes to a new branch after pull

What about:

  1. Branch from the current HEAD.
  2. Make sure you are on master, not your new branch.
  3. git reset back to the last commit before you started making changes.
  4. git pull to re-pull just the remote changes you threw away with the reset.

Or will that explode when you try to re-merge the branch?

What is the best way to modify a list in a 'foreach' loop?

The best approach from a performance perspective is probably to use a one or two arrays. Copy the list to an array, do operations on the array, and then build a new list from the array. Accessing an array element is faster than accessing a list item, and conversions between a List<T> and a T[] can use a fast "bulk copy" operation which avoids the overhead associated accessing individual items.

For example, suppose you have a List<string> and wish to have every string in the list which starts with T be followed by an item "Boo", while every string that starts with "U" is dropped entirely. An optimal approach would probably be something like:

int srcPtr,destPtr;
string[] arr;

srcPtr = theList.Count;
arr = new string[srcPtr*2];
theList.CopyTo(arr, theList.Count); // Copy into second half of the array
destPtr = 0;
for (; srcPtr < arr.Length; srcPtr++)
{
  string st = arr[srcPtr];
  char ch = (st ?? "!")[0]; // Get first character of string, or "!" if empty
  if (ch != 'U')
    arr[destPtr++] = st;
  if (ch == 'T')
    arr[destPtr++] = "Boo";
}
if (destPtr > arr.Length/2) // More than half of dest. array is used
{
  theList = new List<String>(arr); // Adds extra elements
  if (destPtr != arr.Length)
    theList.RemoveRange(destPtr, arr.Length-destPtr); // Chop to proper length
}
else
{
  Array.Resize(ref arr, destPtr);
  theList = new List<String>(arr); // Adds extra elements
}

It would have been helpful if List<T> provided a method to construct a list from a portion of an array, but I'm unaware of any efficient method for doing so. Still, operations on arrays are pretty fast. Of note is the fact that adding and removing items from the list does not require "pushing" around other items; each item gets written directly to its appropriate spot in the array.

jQuery selector to get form by name

$('form[name="frmSave"]') is correct. You mentioned you thought this would get all children with the name frmsave inside the form; this would only happen if there was a space or other combinator between the form and the selector, eg: $('form [name="frmSave"]');

$('form[name="frmSave"]') literally means find all forms with the name frmSave, because there is no combinator involved.

if var == False

Since Python evaluates also the data type NoneType as False during the check, a more precise answer is:

var = False
if var is False:
    print('learnt stuff')

This prevents potentially unwanted behaviour such as:

var = []  # or None
if not var:
    print('learnt stuff') # is printed what may or may not be wanted

But if you want to check all cases where var will be evaluated to False, then doing it by using logical not keyword is the right thing to do.

How do I get the total Json record count using JQuery?

The OP is trying to count the number of properties in a JSON object. This could be done with an incremented temp variable in the iterator, but he seems to want to know the count before the iteration begins. A simple function that meets the need is provided at the bottom of this page.

Here's a cut and paste of the code, which worked for me:

function countProperties(obj) {
  var prop;
  var propCount = 0;

  for (prop in obj) {
    propCount++;
  }
  return propCount;
}

This should work well for a JSON object. For other objects, which may derive properties from their prototype chain, you would need to add a hasOwnProperty() test.

"Could not find Developer Disk Image"

Simply updated Xcode. Solved my problem

Creating a SearchView that looks like the material design guidelines

Another way you can achieve the desired effect is to use this Material Search View library. It handles search history automatically and it's possible to provide search suggestions to the view as well.

Sample: (It's shown in Portuguese, but it also works in english and italian).

Sample

Setup

Before you can use this lib, you have to implement a class named MsvAuthority inside the br.com.mauker package on your app module, and it should have a public static String variable called CONTENT_AUTHORITY. Give it the value you want and don't forget to add the same name on your manifest file. The lib will use this file to set the Content Provider authority.

Example:

MsvAuthority.java

package br.com.mauker;

public class MsvAuthority {
    public static final String CONTENT_AUTHORITY = "br.com.mauker.materialsearchview.searchhistorydatabase";
}

AndroidManifest.xml

<?xml version="1.0" encoding="utf-8"?>
<manifest ...>

    <application ... >
        <provider
        android:name="br.com.mauker.materialsearchview.db.HistoryProvider"
        android:authorities="br.com.mauker.materialsearchview.searchhistorydatabase"
        android:exported="false"
        android:protectionLevel="signature"
        android:syncable="true"/>
    </application>

</manifest>

Usage

To use it, add the dependency:

compile 'br.com.mauker.materialsearchview:materialsearchview:1.2.0'

And then, on your Activity layout file, add the following:

<br.com.mauker.materialsearchview.MaterialSearchView
    android:id="@+id/search_view"
    android:layout_width="match_parent"
    android:layout_height="match_parent"/>

After that, you'll just need to get the MaterialSearchView reference by using getViewById(), and open it up or close it using MaterialSearchView#openSearch() and MaterialSearchView#closeSearch().

P.S.: It's possible to open and close the view not only from the Toolbar. You can use the openSearch() method from basically any Button, such as a Floating Action Button.

// Inside onCreate()
MaterialSearchView searchView = (MaterialSearchView) findViewById(R.id.search_view);
Button bt = (Button) findViewById(R.id.button);

bt.setOnClickListener(new View.OnClickListener() {
        @Override
        public void onClick(View v) {
            searchView.openSearch();
        }
    });

You can also close the view using the back button, doing the following:

@Override
public void onBackPressed() {
    if (searchView.isOpen()) {
        // Close the search on the back button press.
        searchView.closeSearch();
    } else {
        super.onBackPressed();
    }
}

For more information on how to use the lib, check the github page.

MySQL SELECT DISTINCT multiple columns

can this help?

select 
(SELECT group_concat(DISTINCT a) FROM my_table) as a,
(SELECT group_concat(DISTINCT b) FROM my_table) as b,
(SELECT group_concat(DISTINCT c) FROM my_table) as c,
(SELECT group_concat(DISTINCT d) FROM my_table) as d

Favicon: .ico or .png / correct tags?

For compatibility with all browsers stick with .ico.

.png is getting more and more support though as it is easier to create using multiple programs.

for .ico

<link rel="shortcut icon" href="http://example.com/myicon.ico" />

for .png, you need to specify the type

<link rel="icon" type="image/png" href="http://example.com/image.png" />

Unfortunately Launcher3 has stopped working error in android studio?

May 2017; I had the same issue, could not even get to apps as it just cycled between starting and stopping. Went into avd settings, edited the multi core (unticked the box) and set graphics to software Gles. It appears to have fixed the issue

Export data from Chrome developer tool

To get this in excel or csv format- right click the folder and select "copy response"- paste to excel and use text to columns.

How to use android emulator for testing bluetooth application?

You can't. The emulator does not support Bluetooth, as mentioned in the SDK's docs and several other places. Android emulator does not have bluetooth capabilities".

You can only use real devices.

Emulator Limitations

The functional limitations of the emulator include:

  • No support for placing or receiving actual phone calls. However, You can simulate phone calls (placed and received) through the emulator console
  • No support for USB
  • No support for device-attached headphones
  • No support for determining SD card insert/eject
  • No support for WiFi, Bluetooth, NFC

Refer to the documentation

ToString() function in Go

I prefer something like the following:

type StringRef []byte

func (s StringRef) String() string {
        return string(s[:])
}

…

// rather silly example, but ...
fmt.Printf("foo=%s\n",StringRef("bar"))

Groovy - Convert object to JSON string

You can use JsonBuilder for that.

Example Code:

import groovy.json.JsonBuilder

class Person {
    String name
    String address
}

def o = new Person( name: 'John Doe', address: 'Texas' )

println new JsonBuilder( o ).toPrettyString()